Сохранен 517
https://2ch.hk/spc/res/137537.html
Домены arhivach.top и arhivach.site временно не функционируют! Используйте домен ARHIVACH.XYZ.
24 декабря Архивач восстановлен после серьёзной аварии. К сожалению, значительная часть сохранённых изображений и видео была потеряна. Подробности случившегося. Мы призываем всех неравнодушных помочь нам с восстановлением утраченного контента!

Тред тупых ответов №18

 Аноним 24/11/14 Пнд 15:29:17 #1 №137537 
14168321570710.jpg
Тред гениальных вопросов о вот этих вот наших галактиках со вселенными.

Спрашиваем то, за что в других местах выдают путёвку в биореактор. Здесь анонимные ученые мирового уровня критически рассмотрят любые гениальные идеи и нарисованные в Paint схемы, ответят на смелые рассуждения будущих светил космонавтики, вступят друг с другом в околонаучный диспут. На этом шутки в оппосте не заканчиваются.

FAQ:
Q: Можно быстрее?
A: Можно упасть в пузырь альбукерке, наса уже почти надула его.

Прошлый тред надули тут: >>127536
Аноним 24/11/14 Пнд 15:39:28 #2 №137540 
>>137537
>упасть в пузырь альбукерке
Ебанный стыд...
Во-первых, Алькубьерре.
Во-вторых, не упасть, а создавать вокруг корабля изнутри (иначе кина не будет).
В-третьих, НАСА искривляет пространство на десятимиллионную часть, контролируя это сверхточными интерферометрами, до самого варп-привода здесь - как до Антарктиды раком.
Аноним 24/11/14 Пнд 18:48:53 #3 №137567 
ПУЗЫРЬ АЛЬБУРЕРКЕ

Нарекаю это новым мемосом спейсача, аминь.

Сопутствующий вопрос: чому НАСА и прочие так ссут, что бы ничего живого из земляшки не попало на другие планеты? Читал помнится о том, как стерилизируют всякие марсоходы и зонды, это просто пиздец насколько все строго. Что плохого в том, что бы бактерии попали на другую планету/спутник? 99.99999999 % из них подохнет в космосе еще, остальные все равно не смогут адаптироваться. А даже если и адаптируются, это же будет шин и человек, по сути, докажет возможность существования жизни на других планетах плюс, де факто, колонизирует одну из них.
Поясните, спейсаны, ибо яннп.
Аноним 24/11/14 Пнд 18:53:47 #4 №137573 
>>137567
Вроде недавно на поверхности МКС вполне живой планктон находили.
Аноним 24/11/14 Пнд 19:04:59 #5 №137581 
>>137540
Напомните, что за пик был в оригинальном посте?
sageАноним 24/11/14 Пнд 19:07:36 #6 №137585 
>>137573
ШОК СЕНСАЦИЯ НА МАРСЕ НАЙДЕНА ЖИЗНЬ ИСТОЧНИK 2hc.hk/spc/
Аноним 24/11/14 Пнд 19:14:45 #7 №137589 
14168456857900.jpg
>>137585
Зря ты его сагаешь, он прав. В серии проб "Тест", собранных во время ВКД, нашли микроорганизмы.
http://knts.tsniimash.ru/ru/site/Experiment_q.aspx?idE=191
Аноним 24/11/14 Пнд 19:16:50 #8 №137590 
14168458101820.jpg
>>137589
Пикча не та, но похуй.
Мазки одного "Тестов" укладывают на станции.
Аноним 24/11/14 Пнд 19:35:07 #9 №137599 
>>137567
>ПУЗЫРЬ АЛЬБУРЕРКЕ

>Нарекаю это новым мемосом спейсача, аминь
>новым мемом

Ебать ты ньюфажина.
Аноним 24/11/14 Пнд 19:38:43 #10 №137602 
>>137567
Потому что вероятность наличия жизни на том же Марсе небольшая, но отличается от нуля. Что было, когда в Австралию завезли кроликов все хорошо помнят.
Аноним 24/11/14 Пнд 19:40:30 #11 №137604 
>>137602
Я не помню.
Аноним 24/11/14 Пнд 19:44:18 #12 №137606 
>>137589
Я не понял, к чему эксперимент о выживаемости микроорганизмов на внешней стороне МКС к теме о жизни на Марсе?
Аноним 24/11/14 Пнд 19:57:27 #13 №137613 
Если космонавт во время межзвездного перелета подрочит и кончит в космос, какова вероятность, что через n лет малафья попадет на пригодную планету, после чего на ней образуется жизнь?
мимо из /b
Аноним 24/11/14 Пнд 20:08:26 #14 №137617 
>>137613
Нулевая.
Аноним 24/11/14 Пнд 20:14:21 #15 №137621 
>>137606
>>137585
Извини, проебал гринтекст. Это >>137573 была просто заметка к этому
>99.99999999 % из них подохнет в космосе еще, остальные все равно не смогут адаптироваться
Аноним 24/11/14 Пнд 20:23:15 #16 №137626 
>>137567
Так потому что они ищут ту самую жизнь на Марсе. А если найдут ЖИЗНЬ в грязи от собственных траков, то все, исследование провалено. Представь себе историка с машиной времени, или биолога, чихающего в сэмплы.
И швырять наши бактерии для проверки панспермических маняфантазий тоже нужно, но отдельно же.
Аноним 24/11/14 Пнд 21:07:53 #17 №137641 
>>137604
Я тебе вспоминатель принес.
https://ru.wikipedia.org/wiki/Кролики_в_Австралии
Аноним 24/11/14 Пнд 21:19:20 #18 №137648 
>>137641
Спасибо, ты няша. Добра тебе.
Аноним 25/11/14 Втр 19:26:11 #19 №137781 
Насколько твердыми являются нынешние представления о черных дырах? Все телеги, которые пишут в википедии и рассказывают по ББС - это теории, догадки, гипотезы, или 100% инфа?
Аноним 25/11/14 Втр 19:46:01 #20 №137790 
>>137781
Инфы 100% не может быть, так как никто ЧД за влажное вымя не щупал.
Другое дело, что наблюдаемые объекты, кандидаты в ЧД, и эффекты, которые они вызывают, очень точно совпадают с теоретическими моделями этих самых ЧД.
Иными словами: оно выглядит, как утка, ведет себя, как утка и крякает, как утка, следовательно, скорее всего это утка и есть.
>Насколько твердыми являются нынешние представления
Пока все наблюдения не расходятся с теорией, представления вполне себе достоверные.
А так да - ЧД гипотетический объект.
Аноним 25/11/14 Втр 20:52:48 #21 №137804 
>>137567
> 99.99999999 % из них подохнет в космосе еще, остальные все равно не смогут
Это ещё не факт. С «лунным» стрептококком до сих пор разобраться не могут, прожил он эти два года на луне или это уже его при возвращении на Землю в пакет с камерой занесло: http://en.wikipedia.org/wiki/Reports_of_Streptococcus_mitis_on_the_Moon
Аноним 25/11/14 Втр 21:50:10 #22 №137820 
Правда ли что гравитация распространяется моментально? то есть выше скорости света?
Аноним 25/11/14 Втр 22:54:41 #23 №137846 
>>137820
Нынче полагают, что распространяется со скоростью света.
Аноним 25/11/14 Втр 23:23:00 #24 №137867 
>>137846
на основании?
Аноним 25/11/14 Втр 23:47:10 #25 №137880 
>>137867
прост))
Аноним 25/11/14 Втр 23:49:29 #26 №137882 
>>137880
короче ученые сами тольком не знают
Аноним 25/11/14 Втр 23:55:01 #27 №137884 
У меня пара вопросов, наверное будет сумбурно, но я все-таки попытаюсь.
1. Есть ли в Солнечной Системе и где либо еще тела, которые летят по орбите по часовой стрелке, а не наоборот? А вокруг своей оси? Знаю только про Уранус который лежит на боку.
2. Вроде как Земля постоянно теряет вещество, сдуваемое солнечным ветром. Изменяется ли постепенно ее орбита из-за этого и, если да, то каким образом? Уменьшается ли со временем масса Солнца?
3. Как в основном образуются спутники планет, на орбите или в результате столкновений? Почему доминирует мнение, что Луна получилась в результате столкновения чего-то с Землей? Это как-то определили по анализу проб грунта с Луны? Если вещество выбрасывается с поверхности планеты и попадает на орбиту, оно начинает там собираться в более массивное тело, пока не достигнет предела Роша? Вот это все...
4. Почему у спутников планет нет собственных спутников?
Аноним 26/11/14 Срд 01:22:38 #28 №137895 
>>137884
>4. Почему у спутников планет нет собственных спутников?
Если правильно помню, такая херня будет очень нестабильна, центральное тело утянет спутник у спутника лол, и тот шмякнется (скорее всего).
Аноним 26/11/14 Срд 01:35:08 #29 №137901 
Почему нельзя наладить реалтайм-трансляцию с зондав хд, мешает узкий канал связи? Или надо много энергии?
Аноним 26/11/14 Срд 01:36:34 #30 №137902 
>>137895
Таки у одного из спутников Сатурна имхо есть свой спутник, но он обращается на дальней орбите потому Сатурн его не может отобрать
Аноним 26/11/14 Срд 02:05:35 #31 №137907 
>>137884
https://www.youtube.com/watch?v=yJaSHAV4NFM
Аноним 26/11/14 Срд 05:08:15 #32 №137916 
14169676954230.jpg
>>137867
На основании наблюдений и экспериментов.
>>137884
1.В Солнечной системе несколько астероидов имеют ретроградную орбиту и комета Галлея еще. Из 67 извесных спутников Юпитера 50 имеют ретроградную орбиту. У Сатурна примерна половина с ретроградной орбитой.
Среди экзопланет пока обнаружены только две с ретроградными орбитами.
2. Земля теряет не так уж много атмосферы из-за солнечного ветра, более того, она больше теряет атмосферных газов из-за связывания их на поверхности, а вот Солнце - ежесекундно теряет по нескольку миллионов тонн, но учитывая массу Солнца - это тоже не очень много. Так что изменения если и есть, то совершенно незначительны.
3. Единого мнения на это счет нету, есть спутники захваченные из вне, есть спутники образовавшиеся на орбите планет. Какой откуда взялся сказать наверняка довольно сложно. Относительно крупные спутники, в основном, образовались на орбитах планет, тогда как всякая мелочевка бывает чаще всего захвачена.
>Почему доминирует мнение, что Луна получилась в результате столкновения чего-то с Землей?
Потому что у других теорий косяков намного больше, чем у этой.
>Это как-то определили по анализу проб грунта с Луны?
Да, аполлоновцы прилетели с мешком грунта, что бы подтвердить одну из главенствующих на тот момент теорий, а выяснилось, что обосрались все изотопы-изотопчики, пришлось идти чесать затылок снова. И уже после этого была выдвинута модель ударного происхождения Луны, которая тоже имеет свои недочеты, но все равно много лучше, чем то, что было ранее.
>Если вещество выбрасывается с поверхности планеты и попадает на орбиту
Оно там формирует что-то наподобие облака говна, часть из которого выпадает на планету обратно, честь улетает за пределы орбиты планеты, а из оставшегося формируется спутник.
>пока не достигнет предела Роша?
Предел Роша - это то радиус орбиты, на котором спутники разрывает в труху приливными силами. Если спутник вне этого предела он может существовать, как единое целое тело.
>Почему у спутников планет нет собственных спутников?
Потому что орбиты были бы не стабильны. Возможно, что на некоторое время спутники могут заиметь свои спутники, но сколько-нибудь продолжительное время существовать вряд ли могут. Предполагают, что у Реи есть система колец, которую, впрочем, так и не обнаружили. Рея находится достаточно далеко от Сатурна, она достаточно массивна и ее орбита практически круговая, все условия вроде как есть, а вот колец не нашли, хотя по экватору Реи видели лед что как бэ намекает.
Пик - Титан и Тефия
Аноним 26/11/14 Срд 07:19:28 #33 №137921 
>>137901
1. Нет задач. КА должен собрать данные и отправить их пакетом на Землю. Чатиться с аппаратом никому не нужно, да и смысла нет, если сигнал идет несколько минут, а в случае с тем же Вояджером пинг уже больше суток.
2. Земля крутится, а центров дальней космической связи всего несколько штук.
3. КА должен постоянно быть ориентирован антенной в сторону Земли, что далеко не всегда удобно, при сборе данных КА.
4. Канал узкий, да.
5. Тратить энергию, которой и так впритык - тоже глупо, да.
Аноним 26/11/14 Срд 10:14:47 #34 №137946 
Сегодня с утра втентакле увидел репост на стене знакомого:
>Я тут подумал

Вот люди постоянно спорят что лучше, гуманнее и более экологичнее делать с телом перед смертью — предавать его земле, кремировать и так далее. А что если трупы в космос сбрасывать? Тогда за тысячу лет вокруг планеты образуется гигантское кольцо как у Сатурна из не разложившихся из-за отсутствия кислорода трупов. Представьте какая будет крипота.

Спэйсаны, это ж хуйня и никакого бы кольца не было, что бы было? Кроме очевидного вреда для станций, спутников и прочих астронавтов
Аноним 26/11/14 Срд 10:18:37 #35 №137947 
>>137946
Можно выкидывать трупаки на орбиту захоронения, там тела могут оставаться десятками тысяч лет. Но это очень дорого и что бы образовалось заметное кольцо понадобится очень много трупаков. Столько людей просто не наберется.
Аноним 26/11/14 Срд 10:37:43 #36 №137949 
>>137946
В длину человек полтора метра, в ширину - полметра. Если всех людей положить на экваторе, то получится:
40075000 м (экватор) / 1.5 м = 26 716 667 людей на один оборот, кольцо шириной в полметра.
Всё человечество (7млрд) обернет землю 262 раза, т.е. будет кольцо шириной с 131 метр.
На орбите МКС (400км примерно) окружность будет 42689км или 42689000м.
28459334 человека на оборот и 246 оборотов, т.е. 123 метра ширина.
Примерно, как у МКС.
123 метра с расстояния в 400км это очень мало. МКС еле заметная точка. Тут будет едва заметная полоска, многие даже не увидят.
Так как будут мёртвыми на орбите.
Аноним 26/11/14 Срд 10:44:26 #37 №137950 
14169878660380.gif
>>137949
>многие даже не увидят. Так как будут мёртвыми на орбите.

Лол

Алсо, это на орбите МКС, с которой МКС постоянно норовит съехать искупаться в океан.
Аноним 26/11/14 Срд 11:31:13 #38 №137955 
>>137947
А можно кремировать в Солнце. Если долетит.
Аноним 26/11/14 Срд 11:37:23 #39 №137956 
14169910437950.jpg
>>137916
Вот что значит хорошо выспаться. С утра встал, перечитал свои вопросы, и понял, что они и правда тупые. Ну хоть в правильный тред попал.
Аноним 26/11/14 Срд 11:41:01 #40 №137958 
>>137955
Ты хоть представляешь, как сложно запульнуть что-то в Солнце?
Надо почти 30 тысяч км/с скорость развить возле земли.
Для сравнения, самый быстрый аппарат New Horizons набрал 16,26 км/с около Земли.
Космические аппараты, приближавшиеся к Солнцу (для его изучения, например), использовали серии гравитационных манёвров. Стоимости кораблей и миссий составляли сотни миллионов долларов.
Уоррена Баффета и Билли Гейтса так можно похоронить, да. Если они своё состояние на это отдадут.
Аноним 26/11/14 Срд 11:47:00 #41 №137961 
>>137958
Ну не стукай. А же не говорил что это будет дешево. Это будет бесценно.
Аноним 26/11/14 Срд 11:50:43 #42 №137963 
>>137958
> тысяч
Это лишнее по-моему.
Аноним 26/11/14 Срд 12:06:25 #43 №137964 
>>137963
Конечно лишнее. Спасибо за верно замеченную ошибку.
30 км/с (108 000 км/ч). Лего при переводе из км/ч в м/с забыть убрать "к".
Аноним 26/11/14 Срд 18:56:24 #44 №138048 
>>137964
Лего?
Аноним 27/11/14 Чтв 00:24:36 #45 №138165 
>>138048
Технолог.
самопочин
Аноним 27/11/14 Чтв 00:28:28 #46 №138167 
как будет выглядеть секс в космосе?
Аноним 27/11/14 Чтв 00:43:16 #47 №138168 
>>138167
Больше похож на БДСМ, потому что надо будет привязываться.
Аноним 27/11/14 Чтв 00:46:09 #48 №138169 
>>138168
а если не привязываться например
Аноним 27/11/14 Чтв 01:15:47 #49 №138172 
>>138169
Тогда никак. Ну можно в пробирку подрочить.
Аноним 27/11/14 Чтв 02:17:59 #50 №138176 
>>138167
Ну ты берешь ее за ляжки, достаешь писюн, а затем у тебя со слизистых начинает испаряться вся влага, в крови появляются пузырьки воздуха. Через пару секунд теряешь сознание, так и не кончив.
Аноним 27/11/14 Чтв 11:08:58 #51 №138224 
>>138048
Кстати, да. Я Легофаг.

>>138167
Это ОСНЕ неудобно, ведь если ты на МКС, то постоянно дрейфуешь от стенки к стенке. А если ты совершаешь возвратно-поступательные движения тазом, то ещё и дрейфуешь странно. Есть неиллюзорный шанс приложиться головой.

>>138182
А где Сураев? Самый няшный космонавт после Юрия Алексеича ящитаю.
Аноним 27/11/14 Чтв 18:59:14 #52 №138272 
>>138224
>А где Сураев? Самый няшный космонавт после Юрия Алексеича ящитаю.
ух я бы его отняшил, на пару с Алексеичем
Аноним 28/11/14 Птн 02:17:52 #53 №138346 
Я гондон, написал в бамплимитный тред. Перепостирую сюда.

С какой скоростью распространяется гравитация?
Со скоростью света или с более высокой и вплоть до бесконечной?

Разница примерно вот в чём. Если со скоростью света, то, допустим, если Солнце внезапно исчезнет, улетит, будет чем-то сбито и т.д., то до Земли гравитация дойдёт мгновенно или через 500 секунд? Если мгновенно, то Земля дальше полетит по прямой по инерции, а если гравитация дойдёт со скоростью света, то земля ещё 500 секунд будет огибать круг вокруг точки, где было Солнце, а только потом полетит свободно по инерции дальше.

Поняли в чём разница и в чём гипотетический эксперимент как это проверить? Так вот.

Если вы считаете что с бесконечной скоростью, то почему информация не может распространяться быстрее скорости света? Почему нельзя при помощи гравитации передать инфо быстрее? Например, если Земля полетит сразу по прямой или будет лететь 500 секунд по орбите, то траекторию можно замерить не через 500 секунд а сразу. А это уже один из способов передачи информации.

Если же наоборот, гравитация ограничена скоростью света, то почему чёрные дыры притягивают и их можно определить по гравитации? Ведь их не может покинуть свет и электромагнитное взаимодействие. Как и любое поле, распространяющееся со скоростью не выше световой. Получается, гравитация тоже не может их покинуть, а тогда чёрная дыра, даже если бы и существовала, была бы не обнаружима, не имела бы массы, потому что не могла бы распространить гравитацию, затягивая гравитацию в саму себя как и свет. То есть для внешнего наблюдателя чёрная дыра должна скорее была ба быть похожей на пустую область пространства, на войд. Но она имеет массу.
Аноним 28/11/14 Птн 02:35:37 #54 №138364 
Спейсаны, помогите ньюфагу в освоении космоса! Посоветуйте литературу или документалки, с чего стоит начать?
Аноним 28/11/14 Птн 02:36:49 #55 №138368 
14171314090820.jpg
>>138346
>Ведь их не может покинуть свет и электромагнитное взаимодействие. Как и любое поле, распространяющееся со скоростью не выше световой. Получается, гравитация тоже не может их покинуть,


А почему гравитация обязательно должна быть по аналогии со светом чем-то, что может "покинуть"? Типа какая-то материя разлетающаяся кусочками? Кусочки не могут ничего схватить и тянуть к источнику, им в другую сторону надо. Сказал же батька-Эйнштейн, что гравитация - искривление пространства, слушайся. Вопрос про скорость это не отменяет. Или она наоборот, кусочки слетающие со всех сторон к массе и толкающие туда другие массы. Или вечные бесконечные нити, вползающие в массу. А внутри сидит боженька и вяжет носочки.
Аноним 28/11/14 Птн 02:52:21 #56 №138376 
>>138346
>С какой скоростью распространяется гравитация?
Со световой
>или через 500 секунд?
Ничего так просто не исчезает, но вообще да, 500 секунд.
>Если вы считаете что с бесконечной скоростью
Никто ничего не считает и не может. Есть эксперименты и наблюдения, они говорят, что гравитация имеет конечную скорость. По крайней мере, мгновенное распространение оной не наблюдается даже косвенно.
>и их можно определить по гравитации?
До горизонта событий вполне себе может распространяться, за горизонтом событий творится неведомая хуйня. Противоречий нет.

Аноним 28/11/14 Птн 04:35:37 #57 №138414 
>>138368
Ну искривится пространство, ну сожмётся пара астрономических единиц в точку, но эта Хуяня не будет иметь никакой массы. Потому что гравитация, ответственная за массу, не сможет покинуть горизонт событий ЧД. Сама себя затянет в сингулярность и привет.

>>138376
>До горизонта событий вполне себе может распространяться, за горизонтом событий творится неведомая хуйня. Противоречий нет.
Как изнутри горизонта событий оно вылезет, если затянет внутрь быстрее чем успеет распространиться?

Если не вылезет из-под горизонта, то мы никогда не ощутим гравитационного воздействия на нас чёрной дыры. То есть внешний наблюдатель не должен ощущать притяжения, аккреции на какой-то хуежопый диск, да и всех остальных таких вот эффектов.
Аноним 28/11/14 Птн 05:00:50 #58 №138425 
>>138414
>не сможет покинуть

Ты пошёл по кругу. Сказали же тебе, в рамках данной фантазии-теории никому ничего не надо "покидать", не существует никаких штук, которые куда-то улетать хотят. Пространство искривилось вокруг масс не потому, что что-то вылетает из гири или планеты.
Аноним 28/11/14 Птн 06:19:37 #59 №138461 
>>138414
>Потому что гравитация, ответственная за массу
Что это за хуета? Как ты себе представляешь гравитацию, определяющую массу? До такого еще даже самые упоротые струнщики не дошли.
Аноним 28/11/14 Птн 06:54:38 #60 №138466 
Какая гравитация на нейтронных звездах? На сколько время замедляется на них, в сравнение с Землей, насколько они искривляют пространство-время?
Аноним 28/11/14 Птн 16:40:30 #61 №138505 
>>138466
НЗ разные бывают, но вообще такая, что увидишь свой собственный затылок мельком. Там тебя магнитное поле будет волновать не меньше гравитации, думаю.
Аноним 28/11/14 Птн 18:02:32 #62 №138526 
14171869527730.jpg
>>138346
>не могла бы распространить гравитацию, затягивая гравитацию в саму себя
Аноним 28/11/14 Птн 18:06:10 #63 №138527 
>>138466
Такая, что даже если твой труп не разоряет на подлёте и грохнется на поверхность и его, по какой-то фантастической случайности, не разложит на атомы пиздецовой температурой, то его размажет ровным слоем по поверхности НЗ слоем в 1 атом.
Аноним 28/11/14 Птн 18:08:05 #64 №138528 
>>138527
Не разорвет
Аноним 28/11/14 Птн 20:30:18 #65 №138569 
Есть дамп данных с цифрового магнитометра. Предположительно LEMI-30. В начале файла идет XML, дальше бинарные данные. Вопрос, как получить данные в читаемом виде?

пример xml заголовка:
<?xml version="1.0" encoding="windows-1251"?>
<lemi_header version="lemi30i2">
<year>2012</year>
<month>09</month>
<day>03</day>
<base_sampling_rate>256</base_sampling_rate>
<averaging>4</averaging>
<samplingrate>64</samplingrate>
<sensitivity> 2.39300000000000E-0005</sensitivity>
<gain>100</gain>
<bit_to_nT> 5.98250000000000E-0008</bit_to_nT>
<channels>3</channels>
<bytes_per_sample>4</bytes_per_sample>
<one_second_record_size_in_bytes>772</one_second_record_size_in_bytes>
<GPS>
<longitude></longitude>
<lattitude></lattitude>
<altitude></altitude>
</GPS>
<remarks></remarks>
</lemi_header>
Аноним 28/11/14 Птн 20:58:12 #66 №138573 
>>138569
Десериализовать бинарную часть в соответствии с XML-описанием, очевидно же. Это из какого-нибудь публичного датасета файл? Надо курить точные спеки конкретной миссии, ибо датасет может быть упакован как угодно. Обычно с ним какая-нибудь инфа идет, с условиями съема данных, порядком байтов и т.п. XML тоже в каком-то стандартном виде наверняка.
Аноним 28/11/14 Птн 21:38:23 #67 №138580 
>>138573
В инете есть статьи о там, что это обрабатывается матлабом, но алгоритма нигде нет :с Ниче больше нет, только файл с дампом
Аноним 28/11/14 Птн 21:50:14 #68 №138582 
>>138580
Возможно есть какой-то специфический тулбокс для Simulink.
Аноним 28/11/14 Птн 22:30:28 #69 №138596 
14172030288070.png
Спэйсаны, вопрос по микромиру. Понятно, что "увидеть" атомы в микроскоп нельзя. Можно только просканировать и на основе данных смоделировать. И вот мы не можем увидеть даже атом, не то что ядро, но на основе научных методов известно, что атом представляет собой полую структуру с крошечным ядром и сильно удаленными орбитами электронов. Схематическую картинку рисуют в практически любом учебнике по физике для старших классов. У меня вопрос конкретно по нуклонам. Пикрилейтед - "схема" нейтрона, кварковый состав. Однако кварки "по размеру" в тысячи раз меньше стандартного нуклона. Получается, что нейтрон по сути - такая же пустышка, как и атом. И что происходит с конкретными кварками в нейтроне? Они хаотично вращаются, создавая сферическую область, определяемую как нейтрон, или таки упорядочено?
Аноним 28/11/14 Птн 23:09:38 #70 №138614 
14172053783320.png
>>138596
> Они хаотично вращаются, создавая сферическую область, определяемую как нейтрон, или таки упорядочено?

Частицы - это просто волновые функции. Они не вращаются так, как ты себе это представляешь.

Вот так, например, "вращается" электрон вокруг ядра в водороде.
Аноним 28/11/14 Птн 23:12:18 #71 №138616 
>>138596
>И что происходит с конкретными кварками в нейтроне? Они хаотично вращаются, создавая сферическую область, определяемую как нейтрон, или таки упорядочено?
На этот вопрос нет ответа.
>и сильно удаленными орбитами электронов
Орбиталями вообще-то. Схема в учебниках очень упрощенная.
Аноним 29/11/14 Суб 00:08:03 #72 №138636 
>>138614
>Они не вращаются так, как ты себе это представляешь
Да, не представляю, что такое спин так и не понял. Но ведь это же материя. Она же существует в этом трехмерном мире. Ведь имей мы какие-то более точные методы наблюдения, то наверняка можно было бы "пощупать" кварки, несмотря на весь их дуализм? И соответственно создать какую-то наглядную модель для обывателя наподобие атомных моделей.
>Вот так, например, "вращается" электрон вокруг ядра в водороде.
К чему эта картинка без всяких пояснений? Типа "посмотри, как все сложно, и забудь. Твоему разуму это не осилить?" Зачем эти издевки? Я же не наезжаю на современные теории со дна своего невежества. Просто поправь написанное мною.
Аноним 29/11/14 Суб 00:12:29 #73 №138638 
>>138636
>наподобие атомных моделей.
Тебе уже сказали и показали, что атомные модели имеют только общее отдаленное приближение к тому, что есть на самом деле.
>Зачем эти издевки
Это не издевки, это электроны на орбиталях.
Аноним 29/11/14 Суб 00:21:06 #74 №138641 
>>138636
Пояснение к тому пику ин натшел.

http://www.youtube.com/watch?v=OUrFJE60ggo
Аноним 29/11/14 Суб 00:36:34 #75 №138642 
>>138638
>что атомные модели имеют только общее отдаленное приближение к тому
И что с того? На основе несовершенства этих моделей и зародилась квантовая механика. Я лишь спросил о существовании подобных моделей внутреннего строения для нуклонов. И этот>>138616 человек уже ответил.
>это электроны на орбиталях
Хорошо, что такое r,θ,φ? Сферические координаты? Что за цифры в скобках? Значения этих координат? Что означает цветовая шкала? Вероятности? Зачем ты заставляешь меня гадать? Чтоб продемонтрировать нелепость моих догадок?. Очевидно, что картинка рассчитана на "людей в теме". Без конкретных пояснений она никак не противоречит стандартной модели атома водорода в моем сознании.
Аноним 29/11/14 Суб 00:54:36 #76 №138645 
>>138596
А при чем тут астрономия? Такое надо в каком-нибудь наукаче спрашивать.
Аноним 29/11/14 Суб 00:57:26 #77 №138646 
>>138645
>наукач
Я пару раз туда заглядывал, там какой-то пиздец творится.
Аноним 29/11/14 Суб 01:00:41 #78 №138647 
>>138645
Где-то с год назад спросил в аналогичном треде в /sci про достижения современной химии. Что-то про пердеж ответили и больше ничего. Ну а вообще микромир - такая же часть космоса. Описание макрообъектов часто опирается именно на теории микромира.
Аноним 29/11/14 Суб 01:14:48 #79 №138649 
>>138647
>Что-то про пердеж ответили и больше ничего.
Угу, сорт оф балаган.

Этот >>138638 и этот >>138616, это один и тот же.
>Без конкретных пояснений
Конкретное пояснение примерно такое -в этих местах От белого до фиолетового наиболее вероятно нахождение электрона.
Как видишь, со "школьной" версией строения атома вообще совпадений нет.
>подобных моделей
По теории струн, например, взаимодействия и элементарные частицы - это вообще тонкие двумерные вибрирующие струны, какие там модели могут быть.
Аноним 29/11/14 Суб 01:31:48 #80 №138651 
>>138649
>в этих местах От белого до фиолетового наиболее вероятно нахождение электрона
В конкретных моделях на моей памяти тоже нигде не писалось, что электроны буквально вращаются вокруг ядра. И что такое r,θ,φ и цифры в скобках? Если нет конкретных моделей кваркового состава, то я переживу. Просто заинтересовал вопрос, почему нуклоны представлены сферическими моделями.
>По теории струн
Суперсимметрия же почти полностью обоссана экспериментами на БАК. Отвибрировались струны.


Аноним 29/11/14 Суб 04:43:52 #81 №138668 
>>138651
>И что такое r,θ,φ и цифры в скобках?
Сферическая система координат же.
>обоссана
Ты что, в хлеву что ли?
>почему нуклоны представлены сферическими моделями.
Потому что протон, например, имеет условный радиус - который все измерить не могут толком радиус заряда. Собственно не сама форма четкая, как у шарика пинг-понга, а тот радиус, где плотность этого заряда остается на определенном уровне. То есть это не форма протона, как такового, а форма его заряда.
Аноним 29/11/14 Суб 04:50:28 #82 №138671 
14172258282210.png
14172258282221.png
14172258282222.jpg
14172258282233.jpg
>>138668
Смишнявые картинки забыл
Аноним 29/11/14 Суб 06:03:10 #83 №138675 
>>138668
>Ты что, в хлеву что ли
Просто красочный эпитет, ученые-физики и не такие высказывания себе позволяли по поводу не устраивающих их теорий. Свое личное негативное отношение к теории струн я выразил таким способом. Зачем ты цепляешься к словам, когда нужно цепляться к фактам. Хорошо хоть с моделями разобрались, хорошие картинки.
Аноним 29/11/14 Суб 19:22:30 #84 №138734 
14172781502080.png
http://mi3ch.livejournal.com/2760355.html
зачем?
у меня бухурт
sageАноним 29/11/14 Суб 19:43:58 #85 №138742 
>>138734
И что блять? А если бы там позолоченный Будда или Пагода или католический крест был, у тебя бы не было боли, мамкин хейтер?
Ты размазал говно из жжопы по своему лицу и рад.
Аноним 29/11/14 Суб 20:03:25 #86 №138746 
>>138742
была бы, религиоблядки одинаково тупы
Аноним 29/11/14 Суб 20:31:07 #87 №138754 
>>138734
Емнип, Гагарин с Королевым теперь в другом месте висят. Фотку гуглить лень
Аноним 29/11/14 Суб 21:10:03 #88 №138764 
С Аполло-8 библию читали в прямом эфире. Олдрин на Луне причастие совершал, и ниче. А тут иконы повесили, ну охуеть теперь.
Аноним 29/11/14 Суб 21:34:14 #89 №138771 
>>138764
блядь, но зачем, работали без этого всего
Аноним 29/11/14 Суб 22:20:50 #90 №138778 
14172888509550.jpg
>>138734
>>138742
Оба идите нахуй.
Партия сказала: кто не православный - в космос ни-ни. Вот и вешают показушные иконостасы поболее чем у иной деревенской бабки. Линия партии же. Все вопросы к императору и его придворному папе римскому.
А так конечно личное дело, у всех свои божества,талисманы и ритуалы. Даже у советских космонавтов свои загагулины были. Баз Олдрин вобще мессу с Луны служил, и ниче. Ну, кроме батхерта одной упоротой атиеистобляди ЛОЛ.


Аноним 30/11/14 Вск 12:13:43 #91 №138856 
Вот же ебтумать, иконы повесили, суки, фотогрфии перевесили! Религиоблядки! Пидоры!
Аноним 30/11/14 Вск 15:58:21 #92 №138873 
>>138754
И правильно. Иконы на кухне возле толчка и мусорки-Прогресса, Гагарин с Королёвым в рабочей зоне.
Аноним 30/11/14 Вск 16:38:07 #93 №138884 
14173546877570.jpg
Анон, просвети меня пожалуйста:
1. Зачем у союзовской спускаемой капсулы тормозные двигатели, не проще ли было вьебать три парашюта как на апполоне?
2. Почему американские спускаемые модули (маркурий, аполлон и т.д.) садили только на воду? Что бы произошло, если бы что-то пошло не так и аппарат приземлился бы на твёрдую поверхность?
3. Когда орбитальный и приборный отсек союза отстреливаются для входа в атмосферу, как они уводятся от аппарата, чтобы ненароком на столкнутся с ним?
Аноним 30/11/14 Вск 17:00:28 #94 №138895 
>Это положение выбирается с таким расчетом, чтобы к моменту разделения продольная ось корабля была отклонена от направления полета на угол, близкий к 90°. В этом случае после разделения аэродинамические силы не могут вновь вызвать повторное сближение и соударение отсеков.
Аноним 30/11/14 Вск 17:16:32 #95 №138901 
>>138884
1. Экономия веса, мягкость приземления.
2. Потому что тряпки имеют высокую посадочную скорость. То же, что и с Союзами при не выстреливших тормозах.
3. Усилием декаплера.

Вообще, то же самое правило действует на станции - если надо отбросить массу (например запустить микроспутник при ВКД), надо делать это в ретрогрейд. Масса, отброшенная в надир/зенит, может приехать на следующем витке обратно, вовне плоскости орбиты - вообще через полвитка. В прогрейд тоже рискованно, ибо она уходит на более высокоэнергетическую орбиту, и позже оттуда съедет.
Аноним 30/11/14 Вск 17:19:33 #96 №138902 
>>138901
>Вообще, то же самое правило действует на станции
Ну, с поправкой на орбитирование естественно, понятно что перед реентри надо отстреливать вбок.
Аноним 30/11/14 Вск 17:30:09 #97 №138906 
14173578095630.jpg
>>138895
Как на моей пикче, я правильно понял?
Вобщем спейсаны, большое спасибо за ваши ответы, ещё последний вопрос вдогонку: зачем на кораблях серии "Восток" была выбрана схема приземления с катапультированием пилота из капсулы? Выглядит как-то переусложнённо чтоли.
Аноним 30/11/14 Вск 17:33:58 #98 №138907 
>>138906
Потому что скорость посадки востока была 10 м\с. А скорость посадки парашютиста 5 м\с. Так безопаснее.
Аноним 30/11/14 Вск 17:39:01 #99 №138910 
14173583417960.jpg
>>138907
А больше парашютов не влепили по той же причине, что в ответе на мой первый вопрос выше, так?
Аноним 30/11/14 Вск 17:44:55 #100 №138913 
Мне кажется или в voar.io гравитация не ньютоновская, а хуй знает какая и корабли постоянно сближаются с планетой?
Аноним 30/11/14 Вск 17:46:19 #101 №138914 
>>138906
Потому что был шанс получить Комарова в первом полёте - торопились.
Аноним 30/11/14 Вск 17:50:34 #102 №138917 
>>138910
Больше парашютов - меньше надежность, больше вес. Ещё при запиле был геморрой с парашютами (не считая Комарова), а с ним еще и по приземлении что-то делать надо
Аноним 30/11/14 Вск 19:34:36 #103 №138939 
14173652765510.jpg
>>138910
Катапультируемое кресло Востока это не только мягкое приземление на ноги, но и система аварийного спасения на старте. Потом САС сделали в виде отстрела корабля и нужда в кресле отпала.
Аноним 30/11/14 Вск 19:55:40 #104 №138942 
Спэйсаны, а как летали аполлоны? Вот набрал он 11 км/с и полетел к Луне. И что, на подлете к Луне тормозные двигатели включались? Или он прям на 11 километрах гравитацией захватывался?
Аноним 30/11/14 Вск 20:10:40 #105 №138944 
>>138942
>И что, на подлете к Луне тормозные двигатели включались?
this
Аноним 30/11/14 Вск 20:17:56 #106 №138945 
>>138942
Апполон вышел на низкую круговую орбиту над Землёй. Потом газанул и орбита стала эллиптической, вытянутой аж до Луны. В верхней точке эллипса рядом с Луной он газанул ещё. Если бы там Луны не было, то он бы стал двигатьтся вокруг Земли, но высоко как Луна. Но Луна была, поэтому он стал крутиться вокруг неё, а относительно Земли описывал этакую спираль.
Выход на любую стабильную круговую орбиту всегда требует минимум двух включений двигателя.
Аноним 30/11/14 Вск 21:05:50 #107 №138959 
>>138942
> Или он прям на 11 километрах гравитацией захватывался?
Там сильно поменьше было, он же тормозился к апогею.
Аноним 30/11/14 Вск 21:06:20 #108 №138960 
>>138945
Только Аполлон, а не Апполон.
Аноним 30/11/14 Вск 21:20:33 #109 №138962 
14173716336870.jpg
B снова вопросы-вопросики
1. Не смог нагуглить, как планировалось отделять лунный корабль от ЛОК в советской лунной программе: вот корабль оттормозил у Луны, а что дальше? Я думал, блок Д использовался для торможения и отстреливался от всего комплекса вместе с обтекателями ЛК, а уже после сам ЛК отстыковывался и уходил на посадку, но на днях увидел пикчу, где нарисовано отделение от ЛОК и блока и ЛК одновременно, причём блок Д был пристыкован к ЛК, видимо чтобы затормозить его для выхода на траекторию посадки. Где истина, анон?
2. Узнал, что в САС аполлона были некие канарды, что-то вроде маленьких "крыльев" (насколько я понимаю, они нужны были для стабилизации в полёте, если бы САС сработал), но не обнаружил нигде фото их в раскрытом состоянии, только схемы, которые не дают мне чёткого представления, как это дело выглядело бы. В связи с чем, вопрошаю - не найдётся ли у анона фото с выдвинутыми канардами САС аполлона?
Аноним 30/11/14 Вск 22:14:45 #110 №138966 
Почему черные дыры не пожирают всё вокруг себя с адовой скоростью? Раз уж завели разговор про гравитацию, то поясните вот. Всюду почему то показывают просто чд и вокруг неё как ни в чем не бывало тусуются все кому не лень. По идее же чд должна их жрать, становится жирной и упитанной и притягивать все больше вещества и больше пока не станет притягивать другую чд, те сольются и засосут вообще всё.

А также если в центре галактик находятся гигантские чд, то почему они не поглотят эти галактики?
Аноним 30/11/14 Вск 23:32:16 #111 №138972 
>>138966
Если бы объекты были неподвижны относительно ЧД, то так бы и было. Но они движутся и, пытаясь честно упасть на дыру, с разгона пролетают мимо. Кто совсем близко пролетел - становится аккреционным диском.
Аноним 01/12/14 Пнд 00:24:50 #112 №138984 
14173826902340.jpg
>>138962
1. Блок Д нужен для торможения на посадке. В самом ЛК есть только один двигатель - блок Е, посадку и взлет как LM он не потянет.
2. Пикрелейтед.
Аноним 01/12/14 Пнд 00:53:20 #113 №138989 
14173844009300.jpg
>>138984
Большое спасибо тебе, анон
Аноним 02/12/14 Втр 03:10:18 #114 №139203 
>>138966
Спэйсаны, а вот в этих коллайдерах разгоняют комки частиц и сталкивают их. И что это получается? В этих туннелях технический вакуум? На протяжении всех километров?
Аноним 02/12/14 Втр 03:54:44 #115 №139204 
14174816850040.jpg
14174816850061.jpg
>>139203
Вакуум там в трубках, по которым разгоняют частицы. Да, во всех 27 км основного кольца и малых колец тоже. Вакуум примерно такой же, как в космосе. Ищут утечки и трещины даже самых милипиздрических масштабов.
Внутри труб ниобий-титановые магниты, охлаждаемые до 1.9 К гелием, которые позволяют выдать поле, напряженностью в 8 Тл.
Аноним 02/12/14 Втр 05:24:37 #116 №139206 
>>139204
Да, так и думал, что в каких-нибудь маленьких трубочках их разгоняют. Просто не понятно было, нахуя тогда такие широки тоннели. Для обслуживания трубочек видимо.
Аноним 02/12/14 Втр 06:01:58 #117 №139207 
>>139206
Монтаж\демонтаж, обслуживание.
Аноним 02/12/14 Втр 09:51:52 #118 №139227 
14175031128520.jpg
Глупый вопрос: Зачем тащить разгонный блок вместе со спутником на целевую орбиту, а потом на остатках топлива на орбиту захоронения, вместо того, чтобы отбросить его назад (или спутник вперед, что то же самое)?

Конечно эффективность системы отброса ниже, чем ЖРД, но ведь РБ это мертвая масса, которая и так уже выведена, и с которой можно обменяться моментом. Заодно избавляемся от мусора.
Аноним 02/12/14 Втр 10:10:22 #119 №139228 
>>139227
Спуск с высоких орбит энергозатратен и поэтому не выгоден. Одним пинком обратно не улетишь, а космического мусора и так до жопы. Забраться на орбиту захоронения гораздо проще, там всего на 200 км выше ГСО.
Аноним 02/12/14 Втр 12:35:02 #120 №139238 
почему не делают модули космических станций из топливный баков последней ступени? вроде как блохастая скайлеб была так построена. и еще, поясните, насчет надувных модулей.
Аноним 02/12/14 Втр 12:50:26 #121 №139241 
>>139238
А что пояснить насчет надувных модулей? В разработке, в ближайшее время будут испытания, при жизни вероятно увидим космические станции из надувных конструкций.
http://www.bigelowaerospace.com/

Модули не делают, вероятно, потому что нахуй надо - все эти конструкторские работы в космосе. Сейчас баки - просто дешёвые бочки, а нужно будет делать дорогую йобу, вынимать волноломы, трубы. Всё это увеличивает риск наебнуться на любом этапе. Скайлэб, как я понимаю, взлетал уже в таком виде, то есть бак не использовался по прямому назначению. Тип, мощная ракета оставалась вот и заюзали.
Аноним 02/12/14 Втр 13:27:22 #122 №139250 
Можно ли испарять мелкий космолом лазором? Превращать в облачко безобидных частиц мощным лучом за миллисекунду.
Аноним 02/12/14 Втр 13:30:03 #123 №139251 
>>139250
>Можно ли испарять мелкий космолом лазором?
Теоретически да.
>мощным лучом за миллисекунду.
За миллисекунду - нет, да и дороговато.
Аноним 02/12/14 Втр 16:57:35 #124 №139278 
>>139250
Не при текущих технологиях. И невыгодно в любом случае. Кроме того, технология ну очень двойного назначения.
Аноним 03/12/14 Срд 11:34:02 #125 №139329 
>>139238
На водороде не летают, а остальное воняет. И >>139241 прав, конечно.
Аноним 03/12/14 Срд 13:03:53 #126 №139361 
14176010330320.jpg
14176010330341.jpg
Объясните мне, почему происходит обледенение самолетов в полете?
Ведь КА наоборот при входе в атмосферу разогревается, значит по идее от сопротивления с атмосферой самолет тоже должен разогреваться.
Аноним 03/12/14 Срд 13:09:10 #127 №139362 
>>139361
Бля ну ты сравнил дозвук и первую космическую.
Аноним 03/12/14 Срд 13:27:02 #128 №139368 
>>139362
ну т.е. по идее самолет тоже должен разогреваться, только не так сильно?
Аноним 03/12/14 Срд 13:38:57 #129 №139371 
14176031373920.jpg
>>139368
Сильно нагреваются только очень быстрые самолеты типа SR-71 или МиГ-25, потому сделаны из термостойких металлов. Ну как сильно, 400 градусов это хуйня по сравнению с тысячами градусов у КА.
Аноним 03/12/14 Срд 14:07:56 #130 №139389 
>>139371
Ну какбы не хуйня. При 400 уже начинают терять прочность абсолютное большинство конструкционных материалов, алюминию вообще пиздец приходит. Ну кроме еба титановых сплавов и углеродных композитов, но это дорого и муторно.
Аноним 03/12/14 Срд 14:34:16 #131 №139403 
14176064567130.jpg
>>139389
Анон клонит к тому, что дозвуковые пассажирские авиалайнеры и скоростные разведчики и перехватчики - это не одно и тоже, особенно по сравнению со спускаемыми аппаратами - вообще хуйня.
Аноним 03/12/14 Срд 17:32:04 #132 №139437 
>>139403
сколько это будет для 900 км/ч на 10 тыс км при минус 40 градусах?
Аноним 03/12/14 Срд 17:35:40 #133 №139438 
>>139437
На 10 тыс. км тебя будут волновать совсем другие вопросы.
Аноним 03/12/14 Срд 18:07:42 #134 №139443 
>>139438
>>139437
сорри опечатался. 10 тыс. м конечно
Аноним 03/12/14 Срд 18:38:34 #135 №139450 
>>139437
Меньше 40К будет нагрев, при 900 км/ч при этом только на фронтовых плоскостях и в сухом воздухе - кромки крыльев, нос. Боковые панели будут иметь только 2/3 от этого нагрева.
Аноним 03/12/14 Срд 18:39:27 #136 №139451 
>>139450
можно подогревать самолет керосиновыми горелками
Аноним 03/12/14 Срд 18:47:53 #137 №139454 
>>139451
>можно подогревать самолет керосиновыми горелками

Белые люди давно придумали ПОС
Аноним 03/12/14 Срд 20:43:41 #138 №139472 
>>139389
Хуёво, что у нас кислородная атмосфера. Можно было бы делать корпуса из углерода, он легче и доступнее вольфрама и так-же хуёво плавится.
Аноним 03/12/14 Срд 21:53:20 #139 №139489 
>>139361
На сверхзвуке воздух не успевает обтекать каробль и сжимается, а при сжатии газ нагревается, это вносит больший вклад, чем трение.
Аноним 03/12/14 Срд 22:01:53 #140 №139490 
>>139489
На дозвуке тащемта все тоже самое. Аэродинамический нагрев и там и там - одна хуйня, то есть сжатие. Просто при увеличении скорости, растет и температура, зависимость-то от скорости квадратичная.
Аноним 03/12/14 Срд 22:32:36 #141 №139500 
>>139454
да проще можно. экипаж на лету вылазит на крылья и ходит с керосинками
Аноним 03/12/14 Срд 23:15:09 #142 №139518 
>>139500
На флоте юнгу бы послали с тряпкой натирать поверхности.
Аноним 04/12/14 Чтв 05:04:29 #143 №139546 
Можно ли создать замкнутую систему жизнеобеспечения разумных размеров?
Аноним 04/12/14 Чтв 05:23:06 #144 №139547 
14176597864340.jpg
>>139546
Насколько разумных?
Аноним 04/12/14 Чтв 08:04:48 #145 №139552 
>>139547
А на пике замкнутая? Там что, кислород не потребляется? Ведь если потребляется, то для его синтеза нужен, как минимум, солнечный свет, не?
Аноним 04/12/14 Чтв 08:33:11 #146 №139553 
>>139552
Там потребляется углекислый газ и выделяется кислород, который участвует в разложении отмерших частей растений, от чего снова образуется углекислый газ солнечный свет, конечно нужен.
Аноним 04/12/14 Чтв 08:33:26 #147 №139554 
>>139546
Можно, ничто не мешает. Но никто пока не создал.
Аноним 04/12/14 Чтв 08:37:21 #148 №139555 
14176714415790.jpg
>>139546
> разумных размеров
Понятие расплывчатое, действительно.
Аноним 04/12/14 Чтв 14:07:11 #149 №139572 
>>139546
Биосфера-2 была заебись, пока там все эпически пиздой не поросло. Но размеры все равно довольно большие, с собой не унесешь. Да и результаты мягко говоря были так себе.
БИОС-3 был вроде поуспешнее, по крайней мере исследования в нем продолжаются, отчетики можно полистать тут. http://www.ibp.ru/labs/mc.php
Аноним 04/12/14 Чтв 19:35:21 #150 №139852 
>>139572
земля - замкнутая система. а чем проблема её масштабировать
Аноним 04/12/14 Чтв 20:13:22 #151 №139856 
>>139852
Проблемы в основном форс-мажорного характера, ну и просто технические. Ещё несколько подобных экспериментов, и будет успех. Правда, сейчас никому нинужно.

Почитай про Биосферу-2 - прохладное, захватывающее чтиво:
http://www.factroom.ru/facts/26185
Аноним 04/12/14 Чтв 21:18:04 #152 №139863 
Биосфера-2 вообще отличный пример того, как одухотворенное гуманитарное быдлецо теряет свой лоск и человеческий облик.
Аноним 04/12/14 Чтв 21:51:44 #153 №139873 
>Особое внимание уделили полной герметичности куполов, которая превосходила герметичность космических кораблей.
Правда текст там немного по-дебильному написан.

>>139863
Вроде сплошь образованные люди и вообще ИЛ. Да и 150 лямов было вбухано различных специалистов вроде архитекторов и инженеров. На вики написано, что единственные открытия - цемент, связывающий кислород и размножение бактерий, потребляющие этот кислород, но о полученном опыте почему-то умолчали.
Аноним 04/12/14 Чтв 21:55:50 #154 №139874 
Собственно, завязывать надо было на первых серьёзных проблемах - закрывать проект на доработку на пару месяцев и начинать по-новой. А не устраивать сувайвол.

авторитетно заявил
Аноним 04/12/14 Чтв 23:57:15 #155 №139882 
>>139874
Так оно же интересно что там мартышки делать будут. Что характерно, отцы-вдохновители внутрь не полезли . жаль что они в этом куполе друг друга жрать не стали
Аноним 05/12/14 Птн 00:00:46 #156 №139883 
>>139882
Там кто-то пальца лишился. Вроде на работах, но мы-то знаем.
Аноним 06/12/14 Суб 22:56:21 #157 №140356 
14178957814770.png
Большой Взрыв произошёл 13.7 млрд лет назад, так?
Самый дальний наблюдаемый объект находится на расстоянии 13 млрд световых лет, т.е. он там был уже через 700 млн лет после Большого Взрыва.
Возьмём объект с другой стороны; предположим, на расстоянии 8 млрд световых лет. Поскольку мы видим его на таком расстоянии, получается, он там был уже через 5 миллиардов лет после Большого Взрыва при расстоянии в 21 млрд световых лет от него. Выходит, что либо а) объект движется быстрее скорости света либо б) он там был до Большого Взрыва. Поясните, где я неправ.
Аноним 06/12/14 Суб 23:21:40 #158 №140359 
14178973008970.png
>>140356
Аноним 06/12/14 Суб 23:59:47 #159 №140361 
>>140356
"Взорвалось" всё, в том числе и пространство, т.е. материя всегда была относительно равномерно распределена по вселенной. Это раз. Два: пространство продолжает расширятся и расширяется оно не относительно одной точки, а равномерно во всех местах. Поэтому очень отдаленные друг от друга галактики могут двигаться относительно друг друга со скоростью больше скорости света. Напомню, что информация не может передаваться быстрее скорости света, на расширение пространства таких ограничений нет.
Аноним 07/12/14 Вск 00:24:02 #160 №140367 
>>140356
Невозможно определить точку большого взрыва, расслабься.

Вселенная расширяется во всех точках одинаково быстро, соответственно, куда не глянь, всюду объекты удаляются, за редким исключением.

Галактики, например, имеют собственные скорости, которыми они плывут в пространстве, однако само по себе пространство между ними, да и вообще где бы то ни было, вытягивается.

Огромные расстояния обусловлены расширением вселенной, а не разлётом галактик от взрыва, уясни это.
Аноним 07/12/14 Вск 00:25:27 #161 №140369 
>>140367
Хочу добавить, что таким образом большой взрыв сам по себе не совсем взрыв, а точка изначального расширения пространства. Таким образом сам взрыв был везде внутри нашего пространства. Наше пространство как бы и есть тот взорвавшийся "пузырь".
Аноним 07/12/14 Вск 00:47:37 #162 №140370 
А собственно, откуда ученые знают, вселенная расширяется? На основании чего они установили, что галактики разъезжаются в следствии растяжения пространства, а не в следствии того, что они просто разъезжаются?
Аноним 07/12/14 Вск 00:54:20 #163 №140371 
>>140370
Свет, идя от далёких галактик, вытягивается. Видимый диапазон света становится инфракрасным, например. Гамма-свет становится видимым и так далее.

Именно потому новый телескоп Джейм Уэбб инфракрасный по сути, чтобы видеть самые далёкие объекты.
Аноним 07/12/14 Вск 01:02:35 #164 №140372 
14179033557470.jpg
>>140371
Если от расширения пространства даже свет вытягивается, то почему мы не вытягиваемся?
Аноним 07/12/14 Вск 01:08:51 #165 №140373 
>>140372
Потому что наши размеры настолько малы, что расширение пространства внутри нас ничтожно мало и медлительно, скорость этого расширения ничтожно мала. Такое расширение быстро компенсируется притяжением земли, нас самих и так далее.

Потому сами объекты крупнее не становятся, разве что ультракрупные галактики.
Аноним 07/12/14 Вск 01:11:38 #166 №140374 
>>140372
А вообще тебе стоит почитать:
https://ru.wikipedia.org/wiki/Постоянная_Хаббла

Постоянная Хаббла выражается в (км/с)/МПк — это значит, что расстояние в 1 мегапарсек за одну секунду вытягивается примерно на 67,8 км.
Аноним 07/12/14 Вск 02:30:45 #167 №140388 
14179086453460.jpg
Был немало удивлен тем элементарным фактом, что барицентр Солнечной Системы большую часть времени расположен ВНЕ поверхности Солнца.
http://www.youtube.com/watch?v=_cVdI4Qe-wM

Это что же получается, несгораемый корабль из хуйдостаниума может крутить вокруг него совершенно неебические гравиманевры?
Аноним 08/12/14 Пнд 10:48:46 #168 №140545 
>>137537
Спейсач, есть у меня такой вопрос:
Предположим, что имеются две планеты, вращающиеся на одинаковых расстояниях, вокруг одинаковых звёзд. Также, у них примерно одинаковая по составу атмосфераю Одна из них - Земля, вторая же имеет вдвое меньший радиус, массу ~0.75 x 10^24 кг, а также вдвое меньшее ускорение свободного падения (~ 4,9 м/с ^2).
Так вот, вопрос: будет-ли на второй планете атмосфера _вдвое_ более разряжённая, чем на Земле, или ещё боле разряжённая?
Из чего состоит 2-я планета не учитываем, т.к. сферический конь в вакууме.
Аноним 08/12/14 Пнд 12:56:25 #169 №140551 
>>140388
Это центр масс системы, его можно использовать при расчетах движения ВНЕ солнечной системы, на приличном расстоянии. Когда же ты ВНУТРИ системы, тебе важнее локальные флуктуации. Так ты падаешь на Землю, а не на центр масс системы, а твой корабль ебанется прямо в ебаное Солнце, а не обогнет совершенно пустой и бессильный центр масс.

Мимо-хуй-с-горы.
Аноним 08/12/14 Пнд 13:41:11 #170 №140559 
Аноны, поясните за орбитальное движение.

Верно ли что существует взаимосвязь между "высотой" орбиты(апоцентр=перицентр) и скоростью движения космического аппарата?
Т.е. увеличил скорость - перешёл на более крупную орбиту(апоцентр увеличился).

Таким образом выходит что два тела находящиеся на одной идеально круглой орбите, но в разных местах, никогда не столкнутся(расстояние между ними будет всегда одинаковое) ?
Аноним 08/12/14 Пнд 14:33:43 #171 №140572 
>>140545
Летел крокодил в Африку, сколько весит килограмм гвоздей? Плотность атмосферы зависит в первую очередь от количества газа в ней, а потом уже от способности планеты её удержать.

На Венере давление на поверхности 90 с хуем земных атмосфер и даже переходит в суперкритическое состояние (полу-атмосфера-полу-океан), при более-менее сравнимых параметрах планет. На Титане давление больше, чем на Земле, несмотря на то что гравитация у того меньше чем у Луны.

Короче, геология решает. И внешние факторы иногда. (см. >>139538)
Аноним 08/12/14 Пнд 14:39:12 #172 №140573 
>>140559
Все верно.

Только скорость зависит от гравитации тела и полной энергии объекта. Увеличивая большую полуось орбиты (=апоцентр), ты добавляешь объекту полной энергии. Соответственно, перицентр той же высоты он будет проходить на большей скорости, чем раньше. Обратное тоже верно: увеличил скорость в перицентре, поднял апоцентр.
Аноним 09/12/14 Втр 07:13:13 #173 №140754 
>>140545
>вторая же имеет вдвое меньший радиус, массу ~0.75 x 10^24 кг, а также вдвое меньшее ускорение свободного падения
Но так не бывает, ты даешь необходимые данные для расчета g и при этом называешь неправильное значение.
С такими параметрами g на поверхности этой планеты будет около 3 земных g. Экстремальные значения собственного вращения планеты, снижающие 3g до 0,5g, просто невозможны. Иначе она не то, что атмосферу, но и грунт не удержит.
Аноним 10/12/14 Срд 16:11:35 #174 №140934 
Анон, с хуя ли Андромеду называют туманностью, когда это галактика?
Аноним 10/12/14 Срд 16:40:40 #175 №140940 
>>140934
У Гершеля спроси.
sageАноним 10/12/14 Срд 16:41:57 #176 №140941 
14182189179510.jpg
Потому что я так сказал
Аноним 10/12/14 Срд 16:48:34 #177 №140942 
Насколько серьезно в научной среде воспринимаются охуительные истории в духе эксперимента с стареющим/нестареющим близнецами и гравитацией, и червоточины в контексте путешествий и мультиверса? Если несерьезно, то почему это так активно форсится, в том числе и на всяких дискаверях-натгео?
Аноним 10/12/14 Срд 16:57:26 #178 №140943 
>>140942
Ты форточку-то прикрой, а то продует эфиром.
Аноним 10/12/14 Срд 17:07:35 #179 №140944 
>>140942
>Насколько серьезно в научной среде воспринимаются охуительные истории в духе эксперимента с стареющим/нестареющим близнецами и гравитацией
GPS показывает координаты правильно, или существование GPS для тебя - охуительные истории?
>и червоточины в контексте путешествий
Червоточины не противоречат ОТО, но в контексте путешествий - скорее фантастика, чем реальность, что бы червоточины, если существуют, не схлопнулись, нужно слишком много энергии.
>мультиверса
Мультивселенная происходит из многомировой интерпретации квантовой механики. Проверить теорию о Мультивселенной не представляется возможным, в этом смысле она не соответствует строгому т.н. понятию "научности".
Аноним 10/12/14 Срд 17:09:27 #180 №140945 
Скорость вращения планеты влияет на её силу притяжения?
Аноним 10/12/14 Срд 17:27:48 #181 №140950 
>>140945
На силу притяжения не влияет.
Но скорость вращения влияет на вес.
Аноним 10/12/14 Срд 22:09:31 #182 №141000 
Т.е. галактики могут двигаться навстречу друг другу, но за счет расширения вселенной расстояние между ними увеличивается. Так?
Аноним 10/12/14 Срд 22:13:42 #183 №141001 
>>141000
Галактики движутся по направлению к друг-другу, если они гравитационно связаны, например, как Млечный путь и Андромеда, учитывая относительно небольшое расстояние между ними, то скорость сближения превышает скорость расширения пространства. Но если предположить, что где-то далеко есть галактика, которая чисто случайно движется в сторону Млечного пути, то скорость расширения пространства будет быстрее, чем скорость приближения к нам.
Аноним 11/12/14 Чтв 01:52:23 #184 №141022 
14182519434100.jpg
Анон, что это за хреновины и для чего нужны (и "трубки", и этот зелёный штырь)?
Аноним 11/12/14 Чтв 02:50:57 #185 №141030 
>>141022
Зеленая хреновина - это оптический визир-ориентатор. Хуита, куда может посмотреть космонавт и увидеть, что за херня вокруг творится. Типа перископа подводной лодки.
Трубки - часть черной оболочки, вероятно для терморегуляции.
Аноним 11/12/14 Чтв 03:09:32 #186 №141032 
Нагуглилось про трубки. Это кабель-мачты, соединяющие все три отсека. Для разделения предусмотрены пироножи - на сочленениях труб в районе перископа.

http://www.astronaut.ru/bookcase/article/article50.htm
Аноним 11/12/14 Чтв 03:27:28 #187 №141033 
>>141032
Опа, спасибо.
Полчаса назад шуршал на этом сайте, но схему прослоупочил
Аноним 11/12/14 Чтв 04:28:31 #188 №141035 
>>141032
Премного благодарен
Аноним 11/12/14 Чтв 05:27:26 #189 №141039 
>>140950
Вес планеты?
Аноним 11/12/14 Чтв 05:34:38 #190 №141040 
>>141039
Вес тела на планете. Если скорость вращения планеты ускорится, то вес тела уменьшится.
Поскольку ты эталонный гуманитарий, то рекомендую загуглить школьный курс физики и прочитать, чем вес отличается от массы.
Аноним 11/12/14 Чтв 06:38:58 #191 №141041 
14182691383960.jpg
Посаны, являются ли приливные электростанции работающие на притяжении Луны сорт оф вечными двигателями ?

Я понимаю что Луна со временем отдаляется от земли, но представим, что у нас сферическая луна в вакууме, которая находится на постоянном удалении от Земли в таком случае можно ли черпать энергию из приливных электростанций в каком-нибудь искусственном озере из жидкого кислорода даже когда потухнет Солнце ?

Аноним 11/12/14 Чтв 06:49:53 #192 №141043 
>>141041
>представим, что у нас сферическая луна в вакууме
А у нас разве Луна не сферическая в вакууме?
>можно ли черпать энергию из приливных электростанций в каком-нибудь искусственном озере из жидкого кислорода даже когда потухнет Солнце?
Ну приливные электростанции существуют сейчас, так что какая разница, светит ли при этом сферическое Солнце в вакууме, или уже нет.
>искусственном озере из жидкого кислорода
Только найдем материалы, которые не рассыпаются со звоном, при -180 Цельсия и сразу запилим, а че.
>
И еще не плохо было бы не сгореть при расширении оболочки Солнца. Вот это первостатейная задача, я бы сказал.
Аноним 11/12/14 Чтв 06:58:12 #193 №141044 
>>141043
Ну я к тому, что нельзя ли вырабатывать энергию с помощью гравитации спутников, те же приливные электростаниции или можно запилить ветряные мельницы в атмосфере газовых гигантов и избежать неизбежной гибели всего при термодинамической смерти вселенной ?
Аноним 11/12/14 Чтв 07:30:11 #194 №141045 
С дивана кукарекну, что брать энергию от орбитальных сил законами физики не возбраняется, энергия действительно будет забираться. Смерти избежать не удастся, просто человечество пока что так и не научилось толково и дёшево извлекать энергию из таких источников как Луна и Солнце, сжигая углеводороды вместо этого.
Аноним 11/12/14 Чтв 09:08:12 #195 №141054 
>>141044
>гибели всего при термодинамической смерти вселенной ?
Если термодинамическая смерть Вселенной состоится, тут бабушка еще надвое сказала, то никаких атмосфер газовых гигантов не будет, никаких озер из сжиженных газов тоже. Гелий можно будет грызть зубами.
Аноним 11/12/14 Чтв 09:15:10 #196 №141055 
>>141044
Сценарий тепловой смерти подразумевает остывание до средней температуры всей материи. Около 3 Кельвина. Море из жидкого гелия, приливы которого вырабатывают энергию, выглядят максимум невероятно. Проще уж небольшой кораблик поколений запилить с атомным реактором и запасом топлива на пару тысяч лет.
Аноним 11/12/14 Чтв 09:17:36 #197 №141056 
>>141054
>Гелий можно будет грызть зубами.
Но твердого гелия при атмосферном давлении не бывает. Хотя с учетом выпадения этой атмосферы в виде снежка, то и жидкого-то не получится.
Аноним 11/12/14 Чтв 10:15:08 #198 №141061 
>>141044
> избежать неизбежной
Ну конечно можно.

Но только если ты - ахо-сезде
Аноним 11/12/14 Чтв 10:15:48 #199 №141062 
14182821487350.jpg
>>141061
отвалилося
Аноним 11/12/14 Чтв 11:31:54 #200 №141073 
>подразумевает, что везде атмосферное давление одинаковое.
>>141056
Аноним 11/12/14 Чтв 19:42:32 #201 №141161 
>>137537
Вот когда тело падает на черную дыру, то оно разгоняется до скорости света или, по крайней мере, до околосветовых скоростей. А когда оно уже упало в нее, то там действуют квантовые эффекты. Каким образом происходит переход с физики ТО, на квантовую физику?
Тот же вопрос но про физику Ньютона и ТО.
Аноним 11/12/14 Чтв 19:52:43 #202 №141165 
>>140361
>"Взорвалось" всё, в том числе и пространство, т.е. материя всегда была относительно равномерно распределена по вселенной.
То есть, ты хочешь сказать, что материя существует вне вселенной, но ее не существует вне вселенной, т.к. вне вселенной нет пространства? Т.е., вселенная, расширяясь, получает новую материю просто за счет того, что она просто обладает свойством пространства?
Аноним 11/12/14 Чтв 21:46:59 #203 №141208 
>>141161
Квантовые эффекты действуют всегда, но для макрообъектов их воздействие практически нулевое. Также нет никакого перехода от ньютоновской физики к ТО. Если релятивистские эффекты незначительны и ими можно пренебречь, то пользуются ньютоновскими формулами, если нельзя - формулами ТО. ТО более точная и имеет "больший охват", но это не значит, что во вселенной есть какой-то переход.
Аноним 11/12/14 Чтв 22:00:56 #204 №141214 
>>141165
Материя берется из энергии. Но нужны фотоны с колоссальной энергией, сейчас уже таких нет. А вот в первые (нано?)секунды были, тогда-то и образовалась вся материя.
Аноним 11/12/14 Чтв 23:29:19 #205 №141225 
Тред не читал, сразу вопрошал:
Почему в Хаббл или другой телескоп разглядеть всякие туманности типа конской головы, столпов творения и т.д. можно, а ни Луноход ни 146%е пруфы пребывания мурриканцев на Луне нельзя?
Аноним 11/12/14 Чтв 23:43:16 #206 №141228 
>>141225
А что для тебя железобетонный пруф? Ведь конспиролухи на каждый пруф находят контрзаговор. Знаешь, если даже снарядить конспиролуху экспедицию на Луну и показать ему прямо перед носом следы экспедиций аполлонов, то он решит, что это жидорептилоиды прилетели и натоптали специально для него. Есть множество фотографий с орбиты к примеру.
Аноним 12/12/14 Птн 00:18:47 #207 №141237 
>>141228
Ну пожалуй достаточно было бы показать следы и площадку лунного модуля чтобы всех заткнуть. Так нет блять, хотим доказывать по фоткам под каким углом падает тень от Олдрина с американским флагом и Армстронгом. Собственно у меня вопрос именно "почему нельзя?" у всех этих Хабблов дальнозоркость или что?
Аноним 12/12/14 Птн 04:16:19 #208 №141306 
14183469796470.png
>>141237
>Ну пожалуй достаточно было бы показать следы и площадку лунного модуля
Упоротым наркоманам, давно уже показал LRO и площадку и модуль и следы и Луноход, и все что угодно, норкоманы не хотят фоточки, они хотят жрать говно конспирологов, махать ручками и визжать, что все это хуйня и подделка. Смысла метать бисер перед свиньями нет никакого.
>Почему в Хаббл
Потому что у Хаббла зеркало всего 2.5 метра, он не в состоянии различить на Луне объект размером меньше ста метров. Предел разрешающей способности. Физика-хуизика.
Аноним 12/12/14 Птн 04:21:20 #209 №141309 
>>141237
>Ну пожалуй достаточно было бы показать следы и площадку лунного модуля чтобы всех заткнуть. Так нет блять, хотим доказывать по фоткам под каким углом падает тень от Олдрина с американским флагом и Армстронгом. Собственно у меня вопрос именно "почему нельзя?" у всех этих Хабблов дальнозоркость или что?


Никому из нормльных людей это нафиг не надо, американцы были на луне дохуя раз, притащили оттуда кучу грунта и оставили там отражатели по которым только ленивый лазером не пошмалял. Есть даже проект в америках который по этим отражателям смотрит с какой скоростью Луна удаляется.

Но нет всем нужны фоточки.
Аноним 12/12/14 Птн 06:03:24 #210 №141325 
Посаны почему ракеты запускают с земли, если можно поднять их на воздушных шарах километров на 10 и запускать уже оттуда ?
Аноним 12/12/14 Птн 06:55:19 #211 №141327 
>>141325
Это платиновый вопрос?
Аноним 12/12/14 Птн 07:00:46 #212 №141329 
>>141325

Заебешься поднимать. Это тебе не мультик, где человек поднимается в воздух на воздушных шариках.
От 10 км особого профита не будет, да и конструкция, если какой-нибудь сумрачный гений решится ее запилить, получится дохуя неустойчивая. И еще пара десятков минусов у такого решения найдется.
Ждем Космического Лифта. Это будет новая эра в освоении космоса.
Аноним 12/12/14 Птн 07:02:55 #213 №141331 
14183569753270.jpg
Анон, дело в том что есть один Аполлон... В общем, меня настораживает история с аварией Аполлон-13 в пятницу 13. Смущают меня такие совпадения. Есть ли вероятность, что эта авария была для какой-то цели подстроена? Скажем, для привлечения внимания к программе, или для каких-нибудь пропагандистских целей, или ещё чего. Вполне возможно, что это и правда совпадение, но хотелось-бы послушать твоего мнения, анон. Сразу оговорюсь, что:
а) в мистику не верю
б) теории лунного заговора ебал в рот
в) не траль
Аноним 12/12/14 Птн 07:12:13 #214 №141333 
>>141325
1 кубометр гелия поднимает около килограмма массы, теперь возьми калькулятор и подсчитай, какого объема должен быть шар, что бы оторвать от земли ракету, стартовой массой в 700 тонн, потом к этому прибавь массу оболочки и конструкцию, для поддержания и крепления ракеты. Сложи все вместе и по формуле, определи диаметр своего воздушного шарика.
Аноним 12/12/14 Птн 07:21:45 #215 №141334 
>>141331
Дело в том, что было расследование и выяснилось, что виной всему обыкновенное распиздяйство. Подстроить аварию было довольно сложно, так как ни подрядчики, ни заказчики не были в курсе ни точной даты полета, ни то, что проверка узлов и агрегатов будет поручена ебанатам.
Аноним 12/12/14 Птн 07:41:29 #216 №141336 
>>141334
Так там было старое доброе распиздяйство? Я думал банальный отказ техники или что-то в этом роде. Пойду гуглить, а то что я как хуй.
Хотел было сказать, а вдруг небыло самой аварии, типа на самом деле они высадились на Луну и поставили секретный военный зог-излучатель, но почему такая теория не выдерживает критики я уже догадался и сам.
Аноним 12/12/14 Птн 07:43:54 #217 №141337 
>>141336
Да, там выдали тех задание на бак, с изменениями, а подрядчики обосрались маленько, а проверяющие в НАСА обосрались при проверке. В общем насрали все, а разгребал потом ЦУП и астронавты. Хорошо еще не убились насмерть.
Аноним 12/12/14 Птн 07:54:31 #218 №141338 
14183600718760.jpg
>>141337
Человеческое распиздяйство никогда не перестанет меня удивлять, особенно учитывая что в той-же аполлоновской программе измеряли КАЖДЫЙ стежок на скафандрах, а тут умудрились вот так обделаться.
Аноним 12/12/14 Птн 08:03:03 #219 №141339 
>>141338
Люди не совершенны, это ни хорошо, ни плохо. Это просто факт, который нужно учитывать и принимать к сведению. Человек не обосраться не может. Возможно, что это генетически природой заложено и является единственной причиной, почему человек слез с дерева и смог немного в космос.
Аноним 12/12/14 Птн 08:26:47 #220 №141341 
>>141339
Я с тобой полностью согласен, просто у меня бывает бомбит от того, что порой одна простейшая, небольшая глупость (именно глупость, а не ошибка) в итоге сметает потоком говна тысячи человек или результаты огромного труда.
Аноним 12/12/14 Птн 09:04:48 #221 №141342 
>>141331
Еще и старт на 13:13 был назначен.
Аноним 12/12/14 Птн 13:09:13 #222 №141368 
14183789531720.gif
>>141336
Так ведь любой отказ это чей-то проеб. Конструктора, производителя или эксплуатанта. Еще никого в космосе не убила случайность. Другое дело что Аполлон со своими запасами и резервированием был готов к проебам. Вернуться после взрыва на борту это не везение.
Аноним 12/12/14 Птн 13:31:48 #223 №141370 
>>141368
>Так ведь любой отказ это чей-то проеб. Конструктора, производителя или эксплуатанта.
Чем сложнее система, тем сложнее учитывать все факторы, в определенный момент количество становится таким, что все учесть просто не возможно. Есть всякие усталости металлов, в самых неожиданных местах, форсмажоры, в виде упавшего камня с неба, или еще какого явления, которые на момент проеба не были известны.
Но в космосе да, что не авария, то чей-то конкретный косяк, с возможностью назвать конкретную фамилию.
Аноним 12/12/14 Птн 14:14:44 #224 №141380 
>>141370
>>141368
>Еще никого в космосе не убила случайность.
>Но в космосе да, что не авария, то чей-то конкретный косяк

Но ведь в космосе не погиб ни один космонавт: все смерти были в пределах атмосферы Земли. Я чего-то не знаю или вы просто так обобщаете?
Аноним 12/12/14 Птн 14:24:15 #225 №141382 
>>141380
При чем тут, погиб-не погиб. Аварии случались неоднократно.
>все смерти были в пределах атмосферы Земли
И что с того? Техника-то одна и та же. И речь не только об убиенных космонавтах, а об авариях вообще.
Аноним 12/12/14 Птн 16:22:15 #226 №141405 
Обожаю перед сном повтыкать в документальные фильмы/сериальчики про космос.
Смотрел "Через червоточину", "Космическую Одиссею" и еще несколько фильмецов от дискавери.

Что еще есть годного ?
Аноним 12/12/14 Птн 17:13:24 #227 №141422 
>>141405
С Брайаном ко-коКсом есть зачетные сериальчики
Wonders of the Solar System
Wonders of the Universe
Wonders of Life
Human Universe
Если пошукать по интернетам, то в переводе тоже можно найти.
Аноним 12/12/14 Птн 20:49:13 #228 №141526 
>>141405
Тебе наверное не подойдёт, но всем кто интересуется рекомендую Moon Machines в 6 сериях, про всю технику программы аполлон. Жалко про советские программы ничего подобного нет
Аноним 13/12/14 Суб 22:03:52 #229 №141920 
>>137537
Почему был проёбан космос и когда можно ждать новой космической гонки аля 60-е и 2001:Space Odyssey ИРЛ?
Аноним 13/12/14 Суб 22:20:35 #230 №141930 
>>141920
Никогда. Пики ресурсов либо пройдены, либо будут пройдены в ближайшие дни.

А илиткам касмас не упёрся в хуй.
Аноним 13/12/14 Суб 22:47:30 #231 №141937 
14185000508720.jpg
>>141920
Космос не проебан, просто исследования ведутся с помощью роботов.
>и когда можно ждать новой космической гонки аля 60-еc
Никогда, либо очень не скоро. Все заняты новым переделом мира.
>Space Odyssey ИРЛ
Вот это вообще если будет, то достанется пра-правнукам.
Аноним 14/12/14 Вск 09:29:38 #232 №141985 
>>141937
>просто исследования ведутся с помощью роботов
А человеки? Когда будет вторая лунная программа? Колонии на Луне? Высадка на Марс? Возможно ли новую косм. гонку США с КНР?
Аноним 14/12/14 Вск 12:31:32 #233 №142007 
>>141985
Человеки не нужны.
Аноним 14/12/14 Вск 15:19:50 #234 №142025 
>>137537
Возможно ли существование гор/вулканов высота которых больше высоты атмосферы?
Как выглядела бы эта гора в результате эрозии?
Аноним 14/12/14 Вск 15:56:46 #235 №142035 
>>142025
Почему бы не запилить стратовую площадку на пике Эвереста.
+ Высота над уровнем моря около 8500 метров. Можно использовать один двигатель для атмосферы и вакуума. Как знаем больше всего топлива теряется на первых ступенях на подъем пн на высоту, получается мы сохраним оче много топлива.
+ Атмосферное давление в три раза меньше по сравнению с тем же Байконуром. Это намного увеличит удельный импульс ракетного двигателя.
+ Сопротивление атмосферы маленькое. Не нужны мощные обтекатели, а это минус к пн. Автоматические станции можно запускать с оче большим ускорением.
Аноним 14/12/14 Вск 16:03:52 #236 №142038 
>>142035
Среда оче уж враждебная. Нет строительных технологий. Космодром - это ещё и инфраструктура. Огромная такая ебанина из металла и бетона. Осторожно поднять построенную ракету на гору - это задача подороже бочки топлива будет. Ну там ещё парк-заповедник наверное какой-нибудь, юнеска озалупится.
Аноним 14/12/14 Вск 16:06:23 #237 №142040 
>>142038
Ну так гора же она сама из бетона состоит. Насколько я знаю сухая ракета не очень много весит можно запилить какой-нибудь элеватор, а топливо и окислитель по трубам.
Аноним 14/12/14 Вск 16:09:27 #238 №142041 
>>142035
Платиновый ответ: орбита - это вбок, а не вверх, поэтому старт с высокой точки несет больше гемора, чем экономии. Атмосферные потери у ракет и так незначительны, даже с уровня моря.

> Можно использовать один двигатель для атмосферы и вакуума.
> Не нужны мощные обтекатели
Плотные слои атмосферы намного выше Эвереста. Не нужны ГО были бы, если бы ракеты хотя бы с 60-70 километров стартовали. Один и тот же двигатель и так используют (Merlin-1C/1D например, или сопловые насадки у некоторых движков); это больше ограничено разницей в требуемой тяге на разных ступенях, чем эффективностью сопла.
Аноним 14/12/14 Вск 16:20:22 #239 №142046 
14185632220730.gif
>>142025
>Возможно ли существование гор/вулканов высота которых больше высоты атмосферы?
Для начала стоит разобраться, что такое 1) атмосфера, и 2) вулканизм. Например атмосферные модели Земли заканчиваются где-то на 1500км, например ГОСТ Р 25645.166-2004, наиболее точные вообще на ~2500км. А вулканы бывают и на телах без атмосферы.
Аноним 14/12/14 Вск 16:26:13 #240 №142047 
>>142046
В общем виде я имел в виду любые геологические образования пик которых находится выше линии кармана. Также интересует что стало бы с этим образованием в результаты воздействия на него в вершине солнечной\звездной радиации, а внизу воздушной эрозии.
Аноним 14/12/14 Вск 16:36:09 #241 №142052 
>>142047
Линия Кармана никак не связана с вулканизмом, и вообще в контексте планетологии совершенно условна.

Насчет эрозии - можешь погуглить научные работы по эрозии Олимпа, например (в Icarus, на архиве). Он внизу находится в более-менее плотных слоях, а вершина фактически в вакууме. Но плотные это довольно условно, на самом деле для земляшки это километров 40 бы было.
Аноним 14/12/14 Вск 20:15:00 #242 №142158 
>>137537
Что будет если Солнце ВНЕЗАПНО исчезнет/перестанет светить? До какой температуры охладится Земля? Что случится с атмосферой? Будут ли пригодны для жизни глубокие пещеры/шахты, дно океанов и т.д?
Аноним 14/12/14 Вск 20:40:47 #243 №142170 
>>142158
все остынет до 3 градусов кельвина
Аноним 14/12/14 Вск 22:10:13 #244 №142206 
>>142158
Получится примерно как с планетами за снеговой линией. Будет долго остывать. Не до 3К конечно, есть источник тепла в ядре, плюс небольшой приливный разогрев. Сильно изменится химия на поверхности. Нет, не будут пригодны.
Аноним 14/12/14 Вск 22:18:41 #245 №142209 
Но ведь можно как в матрице зарыться под землю?
Аноним 14/12/14 Вск 23:41:30 #246 №142231 
14185896906420.jpg
Анон, нагуглил я тут давеча инфу, что на советской орбитальной станции "Алмаз" (на первой по крайней мере) была установлена 23-мм пушка, и стало мне интересно
1. Каким образом огнестрельное оружие приспособили к космическому вакууму? Про компенсацию отдачи движками нагуглил, но интересует как сама пушка могла там стрелять. Ведь обычный огнестрел в космосе работать просто небудет, или я дурак?
2. Насколько вобще оправдано такое оружие в космосе, там же расстояния и скорсти ебовые просто, или планировалось только вбизи по какому-нибудь подло подкравшемуся капиталистическому шаттлу применять?
Аноним 14/12/14 Вск 23:57:32 #247 №142240 
>>142209
Слишком глубоко придётся зарываться.
Аноним 15/12/14 Пнд 00:00:05 #248 №142243 
>>142231
>огнестрел в космосе работать просто небудет
Почему ты пришел к таким выводам, просто интересно.
>или я дурак?
Ну ты заблуждаешься - это точно.
>и скорсти ебовые
Все относительно.

Аноним 15/12/14 Пнд 00:03:54 #249 №142246 
>>142231
>Ведь обычный огнестрел в космосе работать просто небудет, или я дурак?
Ты дурак Пороз содержит в себе все необходимые для горения вещества и прекрасно работает безвоздушной среде. Главная проблемма именно побороть отдачу в невесомости.

>Насколько вобще оправдано такое оружие в космосе, там же расстояния и скорсти ебовые просто, или планировалось только вбизи по какому-нибудь подло подкравшемуся капиталистическому шаттлу применять?
Расстояния не проблема в невесомости у огнестрела нет максимального радиуса действия, есть только время за которое снаряд долетит доцели. Скорости тоже ибо у спутиников приказы не коррекцию траектории выдаются недостаточно оперативно чтобы увернуться от снаяда, да и вообще относительные скорости цели и стрелка могут быть равны нулю
А так да, ЕМНИП именно как оружие ближнего в космических масштабах, естественно боя и планировали применять.
Аноним 15/12/14 Пнд 00:09:00 #250 №142247 
>>142231
>или я дурак?
Довольно типичное заблуждение.
>или планировалось только вбизи по какому-нибудь подло подкравшемуся капиталистическому шаттлу применять
Да, только так. При разнице скоростей больше 100 м/с попасть невозможно даже если бы туда затащили Шилку, потому как отдача. А вот реальный шанс что США подкрадутся и спиздят станцию вместе со всеми секретами тогда еще был. Лет десять назад я еще читал про другие системы, типа небольшой ракеты для отстреливания мелких целей, но это скорее фантастика и с той поры любое оружие в космосе под запретом.
Аноним 15/12/14 Пнд 00:09:42 #251 №142248 
>>142243
>>142246
Перепады температур же, да и смазка стволу нужна, а в космосе она наверняка себя не так как на земле ведёт - по идее очень быстро это дело должно заклинить или вообще треснуть. Плюс, не понимаю как заряжание этой пушки в невесомости происходит.
Аноним 15/12/14 Пнд 00:13:07 #252 №142250 
>>142248
Тебе словосочетание "безоткатное орудие" ничего не говорит? Куда менее требовательное и капризное изделие чем ёба-нарезное орудие.
Аноним 15/12/14 Пнд 00:29:36 #253 №142252 
14185925767530.jpg
Пришлось ещё и просто про огнестрел погуглить, всё понял. Всем спасибо за ответы, спейсаны
Аноним 15/12/14 Пнд 01:41:35 #254 №142259 
>>142246
>в невесомости у огнестрела нет максимального радиуса действия, есть только время за которое снаряд долетит доцели.
Кучность тоже никуда не девается.
Аноним 15/12/14 Пнд 02:38:26 #255 №142270 
Раз уж пошли терки за короткоствол огнестрел в космосе, то такой вопрос: почему военные не мутят военные спутники? За всякие лазоры, конечно, не говорю. Но, допустим, самый простой вариант - спутник с парочкой МРБ. Такая ракета бы отсоединялась от спутника, до того, как у нее врубалось бы зажигание, чтобы носитель не унесло на Плутон отдачей и разгонялась бы небольшим ракетным двигателем. Боеголовка бы летела почти прямой наводкой в цель, еще сильнее разгоняемая гравитацией и, сдается мне, что хуй бы ее перехватили на такой траектории.
Почему не делают? Есть какие-то тех. ограничения, о которых я не знаю, или, может, конвенции какие, что низя ОМУ в космос, или же это просто будет не так пиздато, как я себе представляю?
Аноним 15/12/14 Пнд 03:44:28 #256 №142282 
>>142270
Договор о принципах деятельности государств по исследованию и использованию космического пространства, включая Луну и другие небесные тела

Принят резолюцией 2222 (XXI) Генеральной Ассамблеи от 19 декабря 1966 года


Статья IV

>Государства — участники Договора обязуются не выводить на орбиту вокруг Земли любые объекты с ядерным оружием или любыми другими видами оружия массового уничтожения, не устанавливать такое оружие на небесных телах и не размешать такое оружие в космическом пространстве каким-либо иным образом.

Луна и другие небесные тела используются всеми государствами — участниками Договора исключительно в мирных целях. Запрещается создание на небесных телах военных баз, сооружений и укреплений, испытание любых типов оружия и проведение военных маневров. Использование военного персонала для научных исследований или каких-либо иных мирных целей не запрещается. Не запрещается также использование любого оборудования или средств, необходимых для мирного исследования Луны и других небесных тел.
Аноним 15/12/14 Пнд 04:04:21 #257 №142284 
>>142282
Присоединюсь к вопросу того анона; если-бы какой-нибудь хитрый пидор всё-таки решил запилить такой спутник в обход договора, то:
1. Были бы реальные профиты от такой схемы?
2. Возможно ли понять, что на орбите находится не обычный спутник, а кукан нацеленный тебе в лоб (при дивном условии, что разведки всех стран проебали разработку и запуск такого спутника, а он сам не сильно выделяется на фоне других внешне)
Аноним 15/12/14 Пнд 04:45:56 #258 №142285 
>>142284
>Были бы реальные профиты от такой схемы?
Только при условии, что это значительная группировка спутников, так как если космический носитель будет пролетать над территорией супостата раз в час, а базирующиеся на земле ракеты долетают за 15-30 минут, то идея так себе. А спрятать большую группировку спутников на орбите просто невозможно. К тому же будет необходимо иметь по всей земле станции слежения, что бы иметь возможность передать сигнал об атаке в любое время. Перехватывать ракеты очень сложно, любая ракета, прошедшая активную фазу траектории очень сложная мишень, так что в этом случае дешевле поставить на дежурство несколько наземных МБР, чем пилить дорогущий спутник-носитель.
>Возможно ли понять, что на орбите находится не обычный спутник, а кукан нацеленный тебе в лоб
Теоретически замаскировать спутник-носитель можно будет. Например, придав ему форму, массу, ЭПР существующих моделей спутников. Так же можно достаточно хорошо экранировать боевые заряды, что бы не фонили. Но регистрировать запуск и назначение все равно придется. Достоинства базирования носителей ЯО на орбите довольно сомнительны, а риск и затраты довольно высоки. Стоит так же учитывать, что при реальном замесе вражина первым делом займется уничтожением группировки спутников.
Аноним 15/12/14 Пнд 04:57:23 #259 №142289 
>>142285
>при реальном замесе вражина первым делом займется уничтожением группировки спутников.
Кстати об этом. Их будут ЭМИ-зарядами глушить в случае чего? Спутников же овердохуя болтается, и они не толпами летают, не очень представляю как большую часть из них быстро снести, да и свои не задеть. Хотя, старый план с ведром болтов на орбиту может и норм, после такой-то войнушки не до космоса будет?
Аноним 15/12/14 Пнд 05:07:58 #260 №142292 
>>142289
После ядерной войны-то? Какой там космос, водички бы найти не зараженной попить, уже радость.
>Спутников же овердохуя болтается
Тех же SM-3 у США более сотни.
Аноним 15/12/14 Пнд 22:56:09 #261 №142496 
14186733691910.jpg
утолите мое любопытство аноны. все ракеты летают на керосине или там на водороде или на несимметричном ..... забыл название .ну на крайняк на твердом топливе.а почему бы не заправить баки топливные БЕНЗИНОМ!! 95 или 98 .. гениально жи -- легкое очень энергоемкое топливо !! почему нет?
Аноним 15/12/14 Пнд 23:13:25 #262 №142504 
>>142496
Во-первых керосин керосин дешевле, чем бензин,
Во-вторых, температура его сгорания ниже, В-третьих керосин более стабилен и не детонирует по пустякам,
В-четвертых керосин меньше подвержен расширению и сжатию, в зависимости от температуры
В-пятых керосин менее пожароопасен,
Аноним 15/12/14 Пнд 23:29:47 #263 №142509 
>>142504
>В-третьих керосин более стабилен и не детонирует по пустякам
Но ведь у бензина значительно выше октановое число. Разве оное не есть параметр стойкости к детонации? Разве это не причина использования высокооктанового топлива в двс?
Аноним 15/12/14 Пнд 23:44:26 #264 №142513 
>>142509
>Но ведь у бензина значительно выше октановое число.
Чем у чего? Ты собрался поискать октановое число у керосина?
Аноним 16/12/14 Втр 01:21:41 #265 №142533 
>>142504
1.ненамного дороже. и уж гораздо дешевле несимметричного
2.температура сгорания ниже да но ит температуры вернее от быстроты расширения раскаленных газов зависит мощность ракеты -- бензин жи мощнее это главный плюс! значит а с соплами че нибудь придумать можно - сплав негораемый пустить на сопла
3.с какого перепуга бензину детонировать - он же в баке не с кислородом-окислителем
4.расширение сжатие - ничего не знаю..тоже по моему несущественно
5. не поджигай и не будет пожроопасным
Аноним 16/12/14 Втр 01:22:31 #266 №142534 
>>142513
>Однако октановое число керосина низкое (ниже 50)

https://ru.wikipedia.org/wiki/%CA%E5%F0%EE%F1%E8%ED
Аноним 16/12/14 Втр 01:24:35 #267 №142535 
>>142533
>бензин жи мощнее это главный плюс!

С чего это вдруг ты так решил?
Аноним 16/12/14 Втр 01:48:53 #268 №142552 
>>142534
>Википедия.
>Цитата без источника.
Oh, you!
Аноним 16/12/14 Втр 02:06:36 #269 №142563 
>>142552
Ну погоди, вот еще цитата из википедии
>Окта́новое число́ (от [изо]октан) — показатель, характеризующий детонационную стойкость топлива (способность топлива противостоять самовоспламенению при сжатии) для двигателей внутреннего сгорания. Число равно содержанию (в процентах по объёму) изооктана (2,2,4-триметилпентана) в его смеси с н-гептаном, при котором эта смесь эквивалентна по детонационной стойкости исследуемому топливу в стандартных условиях испытаний.
Это тоже наглый пиздеж? Я просто не знаю, все мои знания об октановом числе базируются на статье из википедии. Но стелят-то гладко и правдоподобно. Нигде про монополию бензина на октановое число не сказано. Ты точно не обосрался?
Аноним 16/12/14 Втр 02:10:48 #270 №142565 
>>142563
Я хуй знает, что ты носишься с этим октановым числом, которая есть условная цифра, подходящая для определенных обстоятельств. Возьми банку, налей в нее бензину, подожди полминуты и сунь туда руку с зажигалкой, когда чиркнешь колесиком, мы узнаем точно, кто обосрался, а кто нет.

Условно говоря да, если залить керосина в бензиновый двигатель с высокой степень сжатия, то высокооктановый бензин не сдетонирует, а керосин да, но я не это имел ввиду в том перчеслительном посте
Аноним 16/12/14 Втр 02:21:40 #271 №142569 
>>142535
раз температура сгорания выше то соответственно и скорость расширения раскаленных газов выше соответственно и мощь.. элементарная логика.. в чем я не прав?
Аноним 16/12/14 Втр 03:01:47 #272 №142580 
>>142565
Ну так я и спросил, может неудачно вышло, но спросил, что же ты подразумеваешь под стойкостью к детонации? В каких случаях керосин менее взрывоопасен, нежели бензин. Мне стало действительно интересно, ведь есть конкретный показатель детонационной стойкости топлива, по которому керосин солидно уступает бензину.
>когда чиркнешь колесиком
Вообще у меня есть подозрения, что кремниевая зажигалка в данном случае приведет к возгоранию или детонации. Думаю, что высекаемая искра есть подвид окисления, протекание которого в безвоздушной среде сомнительно. Также маслянистая среда явно снизит необходимый коэффициент трения между элементами. Сомневаюсь, что даже просто после погружения в бензин, зажигалка с кремниевым запалом загорится. Но я, конечно, могу фатально ошибаться, поэтому проверять не буду. inb4 зиппа!
Аноним 16/12/14 Втр 03:24:17 #273 №142583 
>>142569
>в чем я не прав?
В опыте ракетостроения.
Первые ракеты, еще в 30-х годах тоже делали на бензине, пытались, хотели. Не шмогли, по разным причинам. Годдард, Небель, Цандер, Тихонравов. Все это дрочили, пока не поняли, что слишком высокая температура - тоже плохо, стенки не выдерживают нагрева. Высокая температура требует дорогих решений по тугоплавкости материалов. Это к уже перечисленным недостаткам.

Бензин. Плотность около 0,71 г/см³.
Керосин. Плотность около 0,80 г/см³
Керосина можно больше взять, на такой же объем.
Температура кипения бензина гораздо ниже, чем керосина, соответственно, нужно сильнее наддувать баки, что бы он не кипел при подъеме, опять таки усложнение конструкции.

Один геморрой с твоим бензином, пришлось даже синтин придумывать, лишь бы на этом бензине не летать.

>>142580
В данном конкретном случае, пары бензина взрывоопасны и могут ебнуть даже если ты просто погладил ебучего кота.
http://www.youtube.com/watch?v=5UIyG2QB7IM
Аноним 16/12/14 Втр 10:18:38 #274 №142611 
Посоны, а давайте представим на минутку, что многоразовость взлетела и стоимость килограмма на НОО равна, предположим, $100 (да, я в курсе, что это гораздо круче самых влажных фантазий Маска).
Но ведь запуск - это еще не все. Что изменится в космоиндустрии от дешевых запусков?
Может, станет выгоднее не тратиться на йоба-материалы, а пилить спутники из чугуния и закидывать их на ракетах побольше? Или вместо проектирования сложных автоматизированных систем (у того же Куриосити изрядная часть многомиллиардной стоимости - R&D) пользоваться безволосыми обезьянами?
Или всем будет насрать и они просто сэкономят свои пару десятков миллионов на выводе?
Аноним 16/12/14 Втр 10:31:23 #275 №142613 
14187150836220.jpg
>>142611
>Что изменится в космоиндустрии от дешевых запусков?
Гоминиды смогут в космос почти по-взрослому. Вычеркнешь еще нолик, будет как у Кубрика в Одиссее.
Аноним 16/12/14 Втр 12:15:17 #276 №142639 
>>142206
> Будет долго остывать
Долго это сколько?
Аноним 16/12/14 Втр 12:40:29 #277 №142642 
>>142639
http://naked-science.ru/article/nakedscience/29-07-2013-297 хотя видел недавно другую таблицу, пишут, за пару месяцев средняя температура до -50 снизится, а до -100 за 5 лет.
Аноним 16/12/14 Втр 14:49:44 #278 №142670 
>>142158
>Будут ли пригодны для жизни глубокие пещеры/шахты, дно океанов и т.д?
Насколько сейчас пригодны настолько и будут. Под землей жарко, океаны очевидно не замерзнут пока недра не остынут, а это через несколько миллиардов лет. На поверхности кислород какая-то часть израсходуется на переработку умерших фотосинтетиков, потом все замерзнет и хз сколько уровень будет снижаться до неприемлемого, может миллионы лет.
Аноним 16/12/14 Втр 14:57:33 #279 №142671 
>>142670
>океаны очевидно не замерзнут пока недра не остынут
Глубина океана мало где переваливает за 5000 метров, для воды, обладающей значительной теплопроводностью - это довольно небольшой слой. А горячо только на разломах, а на трех километрах уже температура от нуля до плюс трех.
Сноубол из Земляшки получится довольно быстро, с промерзанием до тех глубин, где водится одна глубоководная мерзость.
Аноним 16/12/14 Втр 16:07:22 #280 №142691 
>>142671
на спутнике юпитера европе тот же водный океан что то ни хрена не промерзает насквозь .. а там такого подогрева от недр как на земле нет. вот и на земле толщина льда не будет критической --до километра максимум. и то вряд ли
Аноним 16/12/14 Втр 16:19:16 #281 №142702 
>>142691
>а там такого подогрева от недр как на земле нет
Там Юпитер греет так, что Земляшке и не снилось.
>вот и на земле толщина льда не будет критической --до километра максимум.
На Европе подозревают лед толщиной в десятки километров.
>до километра максимум.
И откуда этот километр? Почему именно километр? Почему не 500 метров, не полтора километра?
Аноним 16/12/14 Втр 17:58:29 #282 №142746 
>>142583
>30-х годах
это было сто лет назад...почти.сейчас другие технологии другие материалы.сейчас смогут бензин обуздать ..ведь как горит! соляра 1100 примерно горит, а бензин 2100. мощщщ!!
ну плотность поменьше - баки клепать чуть побольше ... зато энергоёмкость бензина (43961 кдж/кг) выше чем энергоёмкость дизельного топлива (42705 кдж/кг) ..
ящитаю пора вернутся к бензиновым рн
Аноним 16/12/14 Втр 18:01:48 #283 №142748 
>>142746
Возвращайся, кто тебе мешает.
Иди учись по профилю, придешь в Энергомаш, станешь конструктором двигателей и всех убедишь, что бензин пизже керосина.
Аноним 16/12/14 Втр 18:03:50 #284 №142750 
>>142702
смотрел научно-популярный фильм ввс - о ледниковом периоде земли 3млрд лет назад..тоже все замерзло и даже на экваторе лед был 10метров толщины но уж никак не 10км
Аноним 16/12/14 Втр 18:07:32 #285 №142752 
>>142750
И что там СОЛНЦЕ ГАСЛО?
Не мели хуйни, будь любезен, лет через 50-100 океан промерзнет до самого дна, кроме все еще теплых мест, где была вулканическая активность.
Аноним 16/12/14 Втр 19:24:50 #286 №142790 
>>142691
Там много аммиака, который работает как антифриз.
Аноним 16/12/14 Втр 20:24:10 #287 №142811 
>>142611
>Что изменится в космоиндустрии от дешевых запусков?
Резко увеличатся маржины безопасной деятельности, снизятся требования к надежности. Вместо одного 100% работающего аппарата будет дешевле запустить несколько из говна и палок. В целом общая стоимость космоса не сильно снизится (кроме околоземной орбиты), но работать станет намного проще.
Аноним 16/12/14 Втр 21:06:17 #288 №142829 
14187531774070.jpg
Анон, поясни мне за задачи и профиты пилотируемой космонавтики на сегодняшний день и ближайшее будущее. Зачем МКС, если она жрёт деньги вагонами, но не даёт каких-либо значимых выгод? Зачем был Шаттл, если так и не вышел на самоокупаемость? Зачем нам Луна и Марс? Или всё держится на том, чтобы не проебать всё, когда космос человечеству станет-таки НУЖОН нам не пришлось с начинать с нуля? Или что-то ещё? Расскажи мне об этом,анон.
Всё равно я мечтаю увидеть первых людей на Марсе
Аноним 16/12/14 Втр 21:47:54 #289 №142852 
14187556744210.jpg
Анон, поясни мне задачи и профиты фундаментальной науки на сегодняшний день и ближайшее будущее. Зачем Большой Адронный Коллайдер, если он жрёт деньги вагонами, но не дает каких-либо значимых выгод? Зачем нам открытие новых частиц? Или все держится на том, чтобы не проебать всё, когда физика человечеству станет-таки ИНТЕРЕСНА нам не пришлось начинать с нуля? Или что-то ещё? Расскажи мне об этом, анон.
Всё равно я мечтаю увидеть страпельку
Аноним 16/12/14 Втр 21:51:57 #290 №142860 
>>142852
Ну ты сравнил. Всякие зонды и спутники исследуют космос/обеспечивают связь и т.д. Зачем в космосе человеки?
Аноним 16/12/14 Втр 22:25:03 #291 №142897 
>>142829
Задач нет, иди спать.
Аноним 16/12/14 Втр 22:31:06 #292 №142910 
>>142897
Тогда зачем?
Аноним 16/12/14 Втр 22:33:43 #293 №142918 
>>142910
Деньги тратить лишние.
Аноним 16/12/14 Втр 22:38:09 #294 №142925 
>>142918
Лишних денег небывает
Аноним 16/12/14 Втр 22:39:48 #295 №142927 
>>142925
Ладно, уговорил
http://www.mcc.rsa.ru/plan.htm
Аноним 16/12/14 Втр 22:43:39 #296 №142929 
>>142927
Ну так бы сразу, ёпт.
Аноним 17/12/14 Срд 00:57:23 #297 №142978 
>>142040
>гора
>из бетона
Дебил? Где ты горы из бетона видел, поехавший?
мимо инженер-строитель
Аноним 17/12/14 Срд 09:25:58 #298 №143018 
>>142978
Ну ладно, не из битона, а из желе зобетона
Аноним 17/12/14 Срд 09:38:30 #299 №143020 
>>142040
> какой-нибудь элеватор
Ты пони, гора не в чистом поле выросла? Вокруг много других гор. Китайцы заебалить тянуть ж/д ветку в тибет. С большой буквы "З", например.
Это не считая, что с горы имеет смысол запускать только супермелкие ракеты с ПН порядка 1кг. Такие ракетки - все твердотопливные.
А твердое топливо по трубе не идет.
Аноним 17/12/14 Срд 21:20:27 #300 №143107 
Аноны, почему считается, что невозможно превысить скорость света?
Аноним 17/12/14 Срд 21:40:16 #301 №143116 
>>143107
Из-за релятивизма. С приближением к скорости света линейный или ньютоновский прирост скорости (типа, 0.1 с + 0.1 с = 0.2 с) начинает сменяться релятивистским (типа, 0.9 c + 0.9 c = 0.99 c, а не 1.8 c). Чтобы разогнаться до 0.9 нужно потратить гораздо меньше энергии, чем до 0.99, с приближением к единице порядки необходимой энергии тоже растут, и при 1 с необходимое количество энергии уходит в бесконечность.

Однажды учёные зафиксировали космическую частицу, разогнавшуюся до 23 девяток после нуля. Назвали её Ебать Мой Хуй Вот Это Блядь Частица.
http://en.wikipedia.org/wiki/Oh-My-God_particle
Аноним 17/12/14 Срд 21:48:13 #302 №143117 
>>143107
Природа так устроена.
Что бы двинуть что-либо до скорости света в вакууме, нужно приложить бесконечное количество энергии.
Аноним 18/12/14 Чтв 01:39:18 #303 №143173 
поясните про кротовые норы. Смотрел кино там рассказывали про пробои в пространстве, показывали лист бумаги, хуйню-малафью. Я вот только не понял, как их можно найти и не распидорасит ли корабль при пролете?
Аноним 18/12/14 Чтв 01:48:53 #304 №143177 
>>143173
Найти почти никак, если кротовые норы и существуют, то скорее всего, они:
1. не долго живущие,
2. нестабильные
3. размером с пинус комара.

В связи с этим, пропихнуть туда корабль, не затратив на поддержание норы овердохуя энергии нет никакой возможности.

Всю эту ебулду по телику про путешествия через норы рассказывают, потому что она занимательна и красива, дух захватывает, в теории возможна но все это очень далекая фантастика.
Аноним 18/12/14 Чтв 02:33:06 #305 №143184 
Анон, когда человеки высадятся на Марс, какая планета будет следующим кандидатом на пилотируемую экспедицию?
Аноним 18/12/14 Чтв 02:35:46 #306 №143187 
>>143184
Церера, потом поедут на Ганимед и Европу.
Аноним 18/12/14 Чтв 02:36:50 #307 №143188 
>>143187
Есть какие-либо наброски проектов таких экспедиций, или это пока слишком отдалённое будущее?
Аноним 18/12/14 Чтв 02:38:15 #308 №143190 
>>143188
>или это пока слишком отдалённое будущее?
Да, никаких проектов нет. Только в научной и не очень фантастике. Просто больше человекам ехать некуда и не за чем.
Аноним 18/12/14 Чтв 02:40:50 #309 №143191 
>>143187
надо сажать на плутон там по любому есть остатки древней цивилизации
Аноним 18/12/14 Чтв 02:42:21 #310 №143193 
14188597414030.jpg
>>143191
Ты опять выходишь на связь, мудило?
Аноним 18/12/14 Чтв 02:54:15 #311 №143195 
>>143193
у тебя баттхерт от того что я знаю правду?
Аноним 18/12/14 Чтв 10:18:44 #312 №143216 
Когда говорят о Большом Взрыве, то описывают его хронологию начиная с каких-то долей секунды и т.д. Вопрос: скорости были околосветовыми, а значит субъективное время для объектов было еще меньше или то, что описывают описывают уже с поправкой на релятивистский эффект? Этот вопрос вообще имеет смысл?
На разных скоростях объекта, скорость света для него будет разной (опять же из-за релятивистского эффекта)?
>>143116
Так и откуда эта частица. Не написано же. Могли бы хоть догадки написать.
Аноним 18/12/14 Чтв 10:44:54 #313 №143219 
>>143216
>с каких-то долей секунды
Там все измеряется в планковских величинах. Все произошло настолько быстро, что даже релятивисткие эффекты не играли бы никакой заметной роли.
Планковская эпоха продолжалась 0,000000000000000000000000000000000000000000054 секунды, а эпоха великого объединения - примерно 0,00000000000000000000000000000000005 секунды, и так далее. Ну ты понял.

Аноним 18/12/14 Чтв 13:17:37 #314 №143226 
Правильно ли я понял, что во время большого взрыва зародились все измерения, в том числе и время? То есть до этого его небыло вообще? Совсем немогу понять что собой представляет его отсутствие - может ли анон мне научно-популярно объяснить хотя-бы в общих чертах или навести на инфу, более-менее понятно описывающую это (в ультрахардкор матан неумею)?
Аноним 18/12/14 Чтв 14:38:29 #315 №143227 
>>143226
Представь, что ты г-сподь б-г и собрался запилить симуляцию вселенной на своей пеке. Врубаешь ты её а на ней оперативки нет. Если что и было задумано - оно не может быть реализовано никак, т.к. негде.
Сотворяешь саморасширяющуюся оперативку и запускаешь комп - теперь есть память в которой можно симулировать частицы и вселенную, т.е. есть пространство-время.
А доставляя оперативку по сей день ты увеличиваешь пространство-время, увеличиваешь Вселенную.
Пользуясь случаем прошу тебя о несметном богатстве и счастье.
Аноним 18/12/14 Чтв 14:44:48 #316 №143228 
>>143226
>Правильно ли я понял
Да, правильно.
>Совсем не могу понять
Это норма. Мы не знаем никакого другого времени, кроме того, которое однонаправлено и всегда идет вперед. Можно поразминаться мыслью, что там, где нет времени, совершенно необязательно иметь причину для чего-то, так как причина и следствие и именно в этом порядке - это свойства однонаправленного времени, идущего в привычную нам сторону. Соответственно нет никакой причины для БВ, например. Можно даже сказать, что БВ просто не мог не случится. Понять это - довольно непростая задача для мозга, но если упорно разминать мозги, то возможно хотя бы просто осознать, что так может быть.

Есть хорошая книга Стивена нашего колясочника Хокинга, называется "Мир в ореховой скорлупке", написана очень простым языком. Доступна для скачивания бесплатно, без SMS. Очень рекомендую, в том числе и просто для общего развития.
Правда там подробно этот аспект не разбирается. Разбирается в других его книгах, но там матан, в той, или иной степени, ну и потребуется определенная подготовка и некоторый объем знаний. Нахрапом не возьмешь.
Еще можно представить такой график: Ось Х - наша стрела времени, нулевая точка - БВ, а множество точек по оси Y - все возможные "ситуации". Беру ситуации, или события в кавычки, так как имеются ввиду не события во времени, к которому мы привыкли Так вот, если вникнуть в это, то получается, что БВ, просто не мог не случится, не запустив стрелу времени. Точнее все "события" случились, но только одно из них привело к БВ и Вселенной, которую мы знаем. Все остальные "события" проигрались мимо нас, так как они не привели к возникновению Вселенной, где мы живем. Такая вот загогулина, помноженная на антропный принцип.
Стоит ко всему этому добавить, что сам вопрос, "что было до БВ", сам по себе довольно бессмысленный, потому что нас там точно не было, как и всех законов природы, что мы знаем. Сравнивать не с чем.
Аноним 18/12/14 Чтв 16:34:50 #317 №143253 
>>143228
>колясочника
Он Ubuntu использует?
Аноним 18/12/14 Чтв 16:42:16 #318 №143255 
>>143219
> Все произошло настолько быстро, что даже релятивисткие эффекты не играли бы никакой заметной роли.
А они вообще тогда существовали, эти эффекты?
>Планковская эпоха продолжалась 0,000000000000000000000000000000000000000000054 секунды, а эпоха великого объединения - примерно 0,00000000000000000000000000000000005 секунды, и так далее. Ну ты понял.
Но это очень большой промежуток времени.
>>143227
То есть, время и есть то, что расширяет пространство?
>>143228
>Стоит ко всему этому добавить, что сам вопрос, "что было до БВ", сам по себе довольно бессмысленный, потому что нас там точно не было, как и всех законов природы, что мы знаем. Сравнивать не с чем.
То есть, мы никогда не узнаем физики, которая была до БВ? Впрочем, это наверное и не имеет смысла, ведь ее эффектов мы не наблюдаем.
Аноним 18/12/14 Чтв 17:07:46 #319 №143259 
>>143195
ты с пруфами или просто попиздеть?
Аноним 18/12/14 Чтв 17:08:27 #320 №143260 
>>143195
ты с пруфами или просто попиздеть?
Аноним 18/12/14 Чтв 19:41:55 #321 №143317 
Вот все эти теории того, что было во время БВ и после него - это все математика и теория же? Ведь даже те же черные дыры не найдены, а то, что в теории должно ими быть считается лишь кандидатами в черные дыры. То есть, насколько все эти открытия делаются на кончике пера, а не у телескопов, ускорителей и т.д.? Вспомнить хотя бы реликтовое излучение, оно было предсказано, нейтрино всякие так далее. Насколько здесь можно вообще обойтись без эксперимента и наблюдения и все открытия делать лишь на бумаге? Просто не понимаю в чем проблема с открытиями? Компьютеры для сложных вычислений есть, какой-никакой математический аппарат тоже создается, так почему столько теорий и не могут создать единую теорию? Денег же даже не надо.
Аноним 18/12/14 Чтв 20:07:51 #322 №143340 
>>143317
потому что никто на самом деле не узнает как оно было.
Аноним 18/12/14 Чтв 22:17:10 #323 №143404 
>>143317
Теория струн, суперструн, суперсимметрии и вся хуйня. Почти пол-века разрабатывали для нее мат-аппарат. Хорошо получалось, сходилось, только вот эксперименты на БАК суперсимметрию не подтверждают. Теперь разрабатывают новые. Не сцы, уж чего чего, а теорий всегда навалом будет.
Аноним 19/12/14 Птн 05:59:44 #324 №143476 
>>143253
Нет, у него винда. Сейчас вроде семерка.
>>143255
С начала эпохи Великого объединения должны были существовать.
>То есть, мы никогда не узнаем физики
Нет, не узнаем.
>>143317
>Денег же даже не надо.
Пхаа, денег не надо, напомнить сколько стоит Хаббл, к примеру, или БАК? Денег нужна прорва, как ты проверять собрался свои теории? Теория не подкрепленная экспериментами и наблюдениями - так себе теория.
Возьмем те же ЧД, вот по теории они были предсказаны очень давно, найти их было затруднительно и еще до 70-х 80-х годов на чернодырочников смотрели снисходительно, как на недоразвитых детей. Потом были теории, что в центрах галактик могут существовать сверхмассивные черные дыры, построены десятки телескопов, детекторов, интерферометров и прочей лабуды за дохулиард денег, что бы можно было наблюдать отдельные звезды в центре Млечного пути и понять, что там и правда находится какая-то ЙОБА, подходящяя под описания СМЧД. А ты говоришь денег не надо.
Что касается мат аппаратов, то они просто так тоже не берутся, нужны результаты экспериментов и наблюдений. Считать слоненков и попугаев в слепую - так себе занятие.
Аноним 19/12/14 Птн 06:20:25 #325 №143479 
>>143216
>Так и откуда эта частица. Не написано же. Могли бы хоть догадки написать.
"У нас что-то поломалось в оборудовании", вероятно - самая очевидная догадка.
Аноним 19/12/14 Птн 06:32:26 #326 №143480 
>>138364
Этого двачую
Аноним 19/12/14 Птн 06:38:35 #327 №143481 
>>143480
>>138364
Попробуйте начать с книжки Хокинга что-то там про Вселенную или сериала Космос.
Аноним 19/12/14 Птн 06:50:37 #328 №143482 
>>143480
Книга:
Высший замысел С. Хокинга.
Краткая история времени С. Хокинга.
Сериал:
Космос: Пространство и время.

Аноним 19/12/14 Птн 07:06:59 #329 №143483 
>>143476
>Что касается мат аппаратов, то они просто так тоже не берутся, нужны результаты экспериментов и наблюдений
Что-то Алику это не помешало нахерачить мат-аппарат ТО вслепую. Подтверждения пошли несколько позже. И да, имеющиеся современные "теории всего" пока не имеют подтверждения, однако адепты оных вполне активно развивают мат-аппарат для них без всяких экспериментов.
Аноним 19/12/14 Птн 07:29:51 #330 №143484 
>>143483
Ага, конечно в слепую. Максвелл свои уравнения из головы брал, основываясь ни на чем. Майкельсон никаких опытов не ставил. Морли не повторял. Опытов Физо не было. Смещение перигелия Меркурия никто не наблюдал и не заметил. Этвёш никаких опытов не проводил.
А Эйнштейн основываясь же ни на чем решил запилить ТО. Ну а че бы не запилить то? Ну носятся они там со своими эфирными ветрами, ну носились бы дальше, ведь никаких экспериментов, по твоим словам не было.

Аноним 19/12/14 Птн 08:14:52 #331 №143489 
>>143484
Ну с таким подходом можно сказать, что все эксперименты для квантовой гравитации уже готовы. Мат-аппарат ОТО и квант-меха относительно крепко стоит на ногах, стандартная модель имеет множество экспериментальных подтверждений. Только непонятно, почему продолжают наугад тыкать в струны и прочие браны.
Аноним 19/12/14 Птн 08:19:34 #332 №143490 
>>143489
>Только непонятно
Это конкретно тебе непонятно, а другим понятно, что ОТО не клеится со квантовой механикой, от слова совсем.
Аноним 19/12/14 Птн 22:28:18 #333 №143619 
Посоны, а возможно ли сейчас построить полностью автономную космическую станцию. Нивелируем деньги, последствие невесомости. То есть проект Биосфера только в космосе. На 100 человек. На 40 лет.
Аноним 19/12/14 Птн 22:40:35 #334 №143620 
>>143619
Нельзя. СЖО нужна, горючка и запчасти. 100% замкнутая СЖО вообще практически неосуществима.
Аноним 19/12/14 Птн 23:00:32 #335 №143622 
>>143619
Мы и на Земле то с этим обосрались, а ты про космоболванку.
Аноним 20/12/14 Суб 01:40:43 #336 №143659 
>>143620
ну солнечная энергия.
>>143622
ну то был первый , детские болезни исправим, а потом полировать. Но как видно никому это не нужно.
Аноним 20/12/14 Суб 02:26:16 #337 №143662 
>>143659
>ну солнечная энергия.
Растение в треде, все в оранжерею!

>>143659
>ну то был первый
Нет, не первый. И на 40 лет точно никто таких опытов не ставил.
Аноним 20/12/14 Суб 07:24:57 #338 №143680 
Допустим, в центре Земли есть полая область и мы поместили туда предмет. Он будет в невесомости?
Аноним 20/12/14 Суб 07:57:17 #339 №143682 
>>143680
Если предположить, что земля гладкий шар без гор, без мест с большей плотностью, то он будет в равновесном состоянии. Но стоит чуть-чуть отодвинуться от центра масс, как его тут же перетянет и он упадет на стенку.
sageАноним 20/12/14 Суб 08:07:23 #340 №143684 
Проблема-то не в энергии, энергию всегда можно найти, преобразовать. Проблема в субстрате. Не бывает полностью замкнутой сжо приличных размеров, компоненты системы всегда куда-нибудь проябываются. Кислород что-то окисляет, фосфор, кальций где-то солями оседает, углерод, что-то в ионообменниках оседает (которые потом нужно пердолить чтобы вернуть это), да все это в конце концов остается в нас, если мы живы и еще физически растем при этом. Поэтому придется тащить с собой запас таблицы Менделеева согласно нашим потребностям. Причем не забыть ничего. Забыли кобальт в систему добавить, нет кобаламинов, в12, мегалобластическая анемия, гроб гроб кладбище.
Аноним 20/12/14 Суб 08:15:04 #341 №143686 
>>143680
Нет, потому что есть Луна и другие тела, и барицентр системы не совпадает с центром Земли. Даже если сумеем запихать в барицентр, то тела движутся и он соответственно тоже, следовательно неустойчив.
Аноним 20/12/14 Суб 17:04:07 #342 №143732 
>>143682
>>143686

Как упадёт и насколько неустойчив? Там может неравномерности будут не сильнее, чем на МКС, летай спокойно от стенки к стенке.
Аноним 20/12/14 Суб 17:34:50 #343 №143735 
>>143684
а что можно извлечь, допустим из марсианского или лунного грунта? возможно при наличии источника энергии (допустим атомного реактора) можно преобразовать его во что-нибудь полезное
Аноним 20/12/14 Суб 18:04:40 #344 №143738 
>>143732
Центробежная сила, как минимум, не даст ему летать от стенки к стенке.
Аноним 20/12/14 Суб 18:33:35 #345 №143741 
>>143732
>насколько неустойчив?
Нулевая устойчивость. То есть, комар крылышком взмахнет, пушинка упадет и т.д., и все.
Аноним 20/12/14 Суб 20:33:00 #346 №143753 
14190967807830.jpg
>>143741
Нулевая устойчивость это отсутствие самоопрокидывания. Насколько комар сдвинет настолько и останется сдвинутым.
Аноним 20/12/14 Суб 20:43:46 #347 №143754 
>>143741
Это всё будет как на комете поскользнуться - упадёшь, но, может, завтра. Или послезавтра.
Аноним 21/12/14 Вск 11:07:44 #348 №143805 
>>143753
>Нулевая устойчивость это отсутствие самоопрокидывания.
Да.
>Насколько комар сдвинет настолько и останется сдвинутым.
Нет.
Аноним 21/12/14 Вск 12:49:58 #349 №143809 
А если Вселенная бесконечна, по теории вероятности по любому должна быть где-то ещё копия нашей планеты?
Аноним 21/12/14 Вск 12:52:32 #350 №143811 
>>143809
>А если Вселенная бесконечна
Вселенная, по крайней мере наша, не имеет границ, но конечна, если ты имеешь в виду мультивселенную, то где-то определенно есть мир очень похожий на наш.
Аноним 21/12/14 Вск 18:35:32 #351 №143872 
>>143811
чисто теоретически, как можно попасть в другую вселенную? хотя бы теоретически
Аноним 21/12/14 Вск 18:36:03 #352 №143873 
>>143872
имею ввиду параллельную
Аноним 21/12/14 Вск 19:02:57 #353 №143876 
>>143872
>>143873
Никак.
Совсем.
Вообще.
Аноним 21/12/14 Вск 19:04:34 #354 №143877 
>>143876
та ни может быть, я в кино видел, там такое проходило
Аноним 21/12/14 Вск 19:07:05 #355 №143878 
>>143877
Попробуй попасть в кино.
Аноним 21/12/14 Вск 19:09:49 #356 №143879 
>>143878
фильм "Последний киногерой" со шварцем
Аноним 21/12/14 Вск 20:58:00 #357 №143904 
1. Я вот не могу понять, почему гравитацию считают самой слабой силой, но действующей на самые большие расстояния, а электромагнитные силы самыми сильными, но действующие на коротких расстояниях? Ведь если гравитация слабее электромагнитных сил, то как же она сжимает нейтронные звезды и черные дыры, где электроны падают на ядра и т.д.?

2. Откуда берется энергия на разгон при помощи гравитации? В том смысле, что она должна откуда-то убывать, раз где-то прибывает. Еще, часто пишут, что при падении в черную дыру объект может развить скорость, если не световую, то близкую или сравнимую. А ведь это колоссальную энергию надо приложить, чтобы достичь такой скорости, если речь о световой, так и вовсе бесконечную.

3. Можно ли считать, что за счет "излучения" гравитационных волн, черные дыры теряют энергию или даже массу?
Аноним 22/12/14 Пнд 01:10:35 #358 №143965 
>>143904
> Я вот не могу понять, почему гравитацию считают самой слабой силой, но действующей на самые большие расстояния, а электромагнитные силы самыми сильными
Попробуй подпрыгнуть, а потом голыми руками расщепить атом, потом доложи, в чем ты преуспел больше.

>что она должна откуда-то убывать, раз где-то прибывает
Она и убывает. Просто это незаметно. Сравни массу, например, Вояджера и планеты Сатурн, сколько энергии приобрел Вояджер, столько же потерял Сатурн.
>А ведь это колоссальную энергию надо приложить
Ну у ЧД энергии до одури, честно говоря.
>3. Можно ли считать, что за счет "излучения" гравитационных волн, черные дыры теряют энергию или даже массу?
Теряют энергию, эта теория, к слову, подкреплена наблюдениями.
https://ru.wikipedia.org/wiki/PSR_B1913%2B16
Аноним 22/12/14 Пнд 06:04:53 #359 №143995 
>>143965
>Попробуй подпрыгнуть, а потом голыми руками расщепить атом, потом доложи, в чем ты преуспел больше.
Но если я сколлапсирую в черную дыру, то легко раздавлю атомы, но не смогу подпрыгнуть.
>Ну у ЧД энергии до одури, честно говоря.
Погоди. Вот тело падает на ЧД. За горизонтом событий оно, теоретически, разгоняется до световой скорости, так? Но ведь для того, чтобы разогнать тело до такой скорости нужно приложить бесконечную энергию? Т.о., что происходит за горизонтом событий? Тело каким-то образом превращается в безмассовые частицы? Но тогда если нет массы, то нет и гравитации и ЧД.
>Теряют энергию, эта теория, к слову, подкреплена наблюдениями.
Благодарю.
Аноним 22/12/14 Пнд 07:08:34 #360 №143996 
>>143995
>За горизонтом событий оно, теоретически, разгоняется до световой скорости, так?
Нет, не так. Теоретически горизонт событий представляет собой границу, пределы которой не может покинуть материя и, следовательно, информация. Из этого следует, что любая теория о состоянии пространства/вещества под горизонтом - нефальсифицируема. Имеет право на жизнь, но ценность имеет на ряду с теориями о границе вселенной или о происходящем до БВ.
И нет никаких предпосылок полагать, что материя при падении за горизонт разгоняется до "с". До околосветовых - да, скорее всего, но в этих сотых и тысячных "с" и скрыта колоссальная разница.
Аноним 22/12/14 Пнд 07:51:40 #361 №143997 
14192239005720.jpg
>>143805
>Нет.

Гуманитария ответ.
Аноним 22/12/14 Пнд 08:26:03 #362 №143998 
>>143662
Солнце не излучает энергию?

И что там за подобные проекты были до Биосфера-2 ?
Аноним 22/12/14 Пнд 09:12:29 #363 №144003 
>>143998
биосфера один, очевидно же
Аноним 22/12/14 Пнд 09:43:53 #364 №144011 
>>143998
>Но если я сколлапсирую в черную дыру
Ты сначала сколлапсируй, потом поговорим.
>За горизонтом событий оно, теоретически, разгоняется до световой скорости, так?
Нет, не так, с чего ты взял?
>Но тогда если нет массы, то нет и гравитации и ЧД.
Лал.
>>143998
>И что там за подобные проекты были до Биосфера-2 ?
БИОС-1
БИОС-2
БИОС-3
Аноним 22/12/14 Пнд 09:51:49 #365 №144014 
>>143998
>Солнце не излучает энергию?
Жрать ты что будешь, солнцеед?
Аноним 22/12/14 Пнд 13:26:59 #366 №144034 
>>144014

Даже не знаю. Вот на Земле откуда еда берётся? Откуда-то её привозят, с Луны там или с Юпитера. И там тоже закажем, прям на станцию.
Аноним 22/12/14 Пнд 13:41:02 #367 №144037 
>>144034
Закажешь пиццы на 40 лет вперед?
Или ты, болезный, будешь выращивать жратву прямо в космосе на несколько человек? Разводить скот, а по выходным разбрасывать навоз на истощившуюся почву?
Аноним 22/12/14 Пнд 15:10:04 #368 №144056 
>>143997
>Гуманитария ответ.
Кукаретика ответ.
Речь о втором случае.
Первый вообще не может называться нулевой устойчивостью, т.к. есть компенсирующая сила.
Аноним 22/12/14 Пнд 22:02:06 #369 №144101 
>>144056
Вот же балбес. Нулевая это шарик на полу.
Аноним 23/12/14 Втр 00:36:58 #370 №144116 
Почему на Венере атмосфера не сжижилась?
Алсо читал про миссию Пионер/Венера-2, там указано что маленькие зонды (90 кг.) не имели парашютов, при этом пишется что один проработал какое-то время на поверхности. Значит ли это, что из-за плотности атмосферы он замедлился настолько, что смог пережить падение?
Аноним 23/12/14 Втр 01:25:08 #371 №144129 
>>144116
>Почему на Венере атмосфера не сжижилась?
Не достаточное давление и высокая температура, с чего бы ей сжиматься?
>Значит ли это, что из-за плотности атмосферы он замедлился настолько, что смог пережить падение?
Да, запас прочности у зонда большой был, повезло.
Аноним 23/12/14 Втр 02:08:09 #372 №144134 
>>144116
> Почему на Венере атмосфера не сжижилась?
Сжижилась. Нижние слои находятся в суперкритическом состоянии, т.е. газ плавно перетекает в жидкость, точного раздела нет. Венера - это планета-океан. Сверхкритический.

> Значит ли это, что из-за плотности атмосферы он замедлился настолько, что смог пережить падение?
Все венерианские аппараты свободно падали, никакой парашют не выдерживает таких условий. Терминальная скорость в нижних слоях у них была 7-8м/с. Тонули, одним словом.
Аноним 23/12/14 Втр 02:09:53 #373 №144136 
>>137621
Ну вот на поверхности МКС не подохли же.
Аноним 23/12/14 Втр 02:11:18 #374 №144137 
>>144134
Вообще парашюты были.
И даже ДИРИЖАБЛИ, паропунк во все поля
Аноним 23/12/14 Втр 02:41:45 #375 №144141 
>>144134
>Сжижилась.
>Венера - это планета-океан
>Все венерианские аппараты свободно падали

Уёбывай. откуда пришел.
Аноним 23/12/14 Втр 03:24:40 #376 №144145 
14192942805190.gif
14192942805191.jpg
>>144141
kak skazhesh

>>144137
Только в верхних слоях, приземление свободное, торможение амортизатором.
Аноним 23/12/14 Втр 03:36:15 #377 №144148 
14192949752170.jpg
>>144145
Не та диаграмма, вот поширше.
Аноним 23/12/14 Втр 03:58:04 #378 №144149 
>>144148
>Вот пошире
Давай еще шире, до 750 К.
Аноним 23/12/14 Втр 04:55:34 #379 №144155 
>>144101
Нулевая устойчивость - это когда нет восстанавливающих сил, спсобных компенсировать приложенную. Т.о., однажды приложенная сила продолжает действовать. А в приведенном примере она еще и увеличивается.
Можешь не отвечать - все уже поняли, что ты тупой.
Аноним 23/12/14 Втр 05:26:33 #380 №144158 
Почему нельзя покинуть горизонт событий? Землю же можно покинуть хоть с постоянной скоростью не более 30м/с. Если у тебя есть двигатели, конечно.
Аноним 23/12/14 Втр 05:37:52 #381 №144159 
>>144158
Потому что первая космическая на горизонте событий равна скорости света. Достигнуть скорости света нет никакой возможности.
Аноним 24/12/14 Срд 07:27:00 #382 №144516 
>>144159
А зачем нужна 1я космическая? Мы же не камень кидаем. Землю же можно покинуть, двигаясь на двигателях с постоянной скоростью, например 100км в час.
Аноним 24/12/14 Срд 07:35:22 #383 №144519 
>>144516
Как тебе еще объяснить, что с горизонта событий съебать нельзя? Внутрь- можно, наружу - нужно превысить скорость света.
Аноним 24/12/14 Срд 08:54:38 #384 №144530 
>>144519
вообще-то скорость света таки превысить можно https://ru.wikipedia.org/wiki/Пузырь_Алькубьерре
Аноним 24/12/14 Срд 09:01:51 #385 №144532 
>>144530
Ой съеби уже с википузией, речь там идет о искривлении пространства, а не о преодолении объектом скорости света.
Аноним 24/12/14 Срд 09:26:06 #386 №144539 
>>144532
тащемта речь о преодолении
Аноним 24/12/14 Срд 10:47:29 #387 №144546 
>>144539
Тащемта иди прочитай про свой пузырь сначала, преодалевальщик. Объект вообще не двигается - сжимается пространство перед объектом и разжимается за ним. Во-вторых неплохо было бы этот пузырь надуть экспериментально, хотя бы на полшишечки и не обосраться.
Аноним 24/12/14 Срд 11:18:55 #388 №144549 
>>144546
>сжимается пространство перед объектом и разжимается за ним
Не слишком ли напряжно всю вселенную передвигать?
Аноним 24/12/14 Срд 11:32:14 #389 №144550 
>>144549
Нет нужды передвигать всю Вселенную, нужно только сравнительно небольшую часть деформировать. Но все равно это очень напряжно, в этом и основная трудность, необходимо слишком много энергии на это.
Аноним 24/12/14 Срд 17:46:06 #390 №144645 
велась ли видеосъемка где-нибудь кроме луны? или только фотали на камеру от мобильника?
Аноним 24/12/14 Срд 22:47:23 #391 №144708 
>>144645
Зойчем?
Всегда можно наебенить фоточек и сделать так:

http://www.youtube.com/watch?v=sNUNB6CMnE8

http://www.youtube.com/watch?v=rFDjAfwmWKM
Аноним 24/12/14 Срд 22:53:17 #392 №144710 
>>144708
а если с поверхности
Аноним 24/12/14 Срд 22:58:20 #393 №144713 
>>144710
А в чем принципиальная разница?
http://www.youtube.com/watch?v=DaVSCmuOJwI
Аноним 24/12/14 Срд 23:52:08 #394 №144727 
14194543288780.jpg
Анон, есть ли на первой/второй ступенях Союза и подобных ракет дросселирование тяги, или на нижних ступенях просто "газ в пол" как в ТРД? Если есть, то зачем оно?
Аноним 25/12/14 Чтв 00:29:46 #395 №144737 
>>144727
тебе нахера это знать?
Аноним 25/12/14 Чтв 01:01:22 #396 №144743 
>>144737
Охуел чтоли, так отвечать?
Аноним 25/12/14 Чтв 01:04:55 #397 №144746 
>>144727
Есть. Для снижения перегрузок при выгорании топлива.
Аноним 25/12/14 Чтв 01:12:36 #398 №144748 
>>144727
вроде как там тяга постоянная
Аноним 25/12/14 Чтв 01:15:32 #399 №144750 
>>144727
зачем они рельсы покрасили? почему не в уставной зелёный?
Аноним 25/12/14 Чтв 01:17:18 #400 №144751 
>>144750
Чтобы было няшно.
Аноним 25/12/14 Чтв 01:20:20 #401 №144754 
>>144750
Это ж не сами рельсы, а возле них полоски чтоб не потерять рельсы
Аноним 25/12/14 Чтв 16:18:03 #402 №144846 
Погодите, если любая частица - это волна как вы тут говорите, то, с учетом постоянной Планка, у волны, а значит и материи, есть предельная частота? А раз так, то вся Вселенная с этой планковской частотой исчезает и воссоздается вновь с периодом в планковское время? А раз так, то, если предположить, что постоянная Планка справедлива только для нашей Вселенной, в промежутке между исчезновением всей Вселенной и ее воссозданием с учетом всех законов, скоростей и вытекающим из них местоположением материи, может уместиться бесконечной количество других Вселенных, причем, из того же самого вещества, что и мы? То есть, пока я писал этот пост я хрен знает сколько раз побывал звездой, газовым гигантом, вакуумом, галактической шлюхой, спасителем Вселенной и даже великим ученым в других Вселенных? А при достижении скорости света за счет эффекта сокращения длины должна сокращаться и планковская длина? Вы наркоманы, что ли?
И еще, вот летит какой-нибудь хер на скорости света к звезде, прямо на нее, ну может чуть левее от центра. За счет того самого эффекта сокращения длины, звезда должна для него сжаться в сингулярность и он пролетит мимо нее? Как быть с ее гравитацией, увеличится, уменьшится, останентся неизменной?
Аноним 25/12/14 Чтв 16:28:02 #403 №144851 
>>144846
>это волна как вы тут говорите
Никто тут этого не говорил, иди проспись, пьянь гидролизная.

Аноним 25/12/14 Чтв 16:32:46 #404 №144852 
>>144851
Я же своими глазами видел, что писали. Вот эти, например, >>138614 >>138649, и еще один прямо так и написал, что частица - это волна, только не могу найти.
Аноним 25/12/14 Чтв 16:39:26 #405 №144854 
>>144852
Там написано, что частица описывается волновой функцией, а не то что частица - это волна. Почувствуй разницу. У этого >>138649 вообще про волну не слова.
>еще один прямо так и написал, что частица - это волна, только не могу найти.
Будет очень странно, если найдешь.
Аноним 28/12/14 Вск 14:53:18 #406 №145345 
А Аллах летает по экваториальной или полярной орбите?
sageАноним 28/12/14 Вск 15:13:44 #407 №145352 
>>145345
Аллах не летает. Он неподвижен. Все летает вокруг аллаха.
Аноним 28/12/14 Вск 19:26:15 #408 №145405 
>>145345
Все сущее разлетается от той точки, где Аллах сотворил вселенную.
Аноним 28/12/14 Вск 21:36:20 #409 №145426 
14197917807310.jpg
Интересно, никто в последнее время не слышал про повторение-продолжение проекта "Знамя"?
Может, у кого есть новости про космические зеркала ? Это ведь как минимум, неплохой способ сделать космос эффектным.
Аноним 28/12/14 Вск 21:42:02 #410 №145430 
>>145426
сжигать в адском пламени города? нет уж, увольте
Аноним 28/12/14 Вск 22:26:44 #411 №145435 
>>145426
Для меня большая загадка, почему такую охуенную идею так никто и не попытался повторить. Для обывателя это буквально самый очевидный профит космической программы, пусть и довольно бесполезный. Простому человеку хоть и объясняют в телевизорах зачем ученые гоняются за астероидами, но результат от этого он не может пощупать. А тут такая-то летающая ёба в небе. Наверняка можно и закраудсорсить такой проект какому-нибудь частному космическому агентству.
https://www.youtube.com/watch?v=w8I25H3bnNw
Аноним 28/12/14 Вск 22:34:53 #412 №145439 
>>145435
На кикстартере видел проект, суть такова: запустить на НОО отражатель чтобы он светил как иридиум только всё время. Типа зажги свою звезду. Не помню чем дело кончилось.
Аноним 28/12/14 Вск 22:51:36 #413 №145443 
>>145439
Вроде ничем, они так и не набрали бабла. А жаль.
Аноним 29/12/14 Пнд 08:42:40 #414 №145504 
Может ли как-нибудь навредить тяжелая заряженная частица с произвольной энергией, если прошьёт тело человека на орбите в произвольной точке? Через мозг если пролетит, например.
Аноним 29/12/14 Пнд 12:15:24 #415 №145516 
>>145504
Вопрос из разряда: может ли пуля как-нибудь навредить общей структуре горы Эверест. Теоретически может, на околосветовой скорости. На практике же таких пуль не существует.
Аноним 29/12/14 Пнд 12:17:34 #416 №145517 
>>145516
Так я и сам себе ответить могу.
Аноним 29/12/14 Пнд 12:21:26 #417 №145518 
>>145517
Ну а какой ты хочешь ответ? Ребята с орбиты вроде пока не жаловались. На, почитай, если можешь в ангельский
https://en.wikipedia.org/wiki/Ultra-high-energy_cosmic_ray
Может что-то интересное найдешь.
Аноним 29/12/14 Пнд 12:32:31 #418 №145519 
>>142289
>план с ведром болтов на орбиту
Проиграл в голос. Что за хитрый план, можно по подробнее?
Аноним 29/12/14 Пнд 12:50:24 #419 №145522 
>>145519
Вы звать ис кус ственный син дром Кес сле ра.
Аноним 29/12/14 Пнд 12:55:26 #420 №145524 
>>145518
Это не то.

https://www.physicsforums.com/threads/can-the-oh-my-god-particle-kill-an-astronaut.422135/

Единственное, что нашёл. Полагаю, лучше и не найду ничего.
Аноним 29/12/14 Пнд 12:59:43 #421 №145525 
>>145522
Спа си бо .
Аноним 29/12/14 Пнд 20:12:46 #422 №145581 
были ли случаи тяжелый психических расстройств на орбите? или космонавты настолько устойчивы что им похер?
Аноним 29/12/14 Пнд 20:31:28 #423 №145584 
>>145581
>космонавты настолько устойчивы

Строгий отбор и подготовка исключают всяких ебанько на орбите. Только няшки-адекваты, способные провести с другими неделю, находясь в позе эмбриона и нюхая чужие потные подмышки.
Аноним 29/12/14 Пнд 20:33:24 #424 №145585 
>>145584
я слышал один чувак на орбите пару лет болтался. русский вроде или хохол
Аноним 29/12/14 Пнд 20:37:17 #425 №145586 
14198746372730.jpg
>>145581
Что есть пси-расстройство?
Вот я щитаю, что это ПГМ и иконы на орбите, например

И вот еще вопрос, не в тему раздела, но вроде здесь даже бывают адекваты.
Как вернуть поле ввода яндекс-капчи в форму для ответов в фаерфоксе? Пишу с оперы, но у меня ломка
Аноним 29/12/14 Пнд 20:38:50 #426 №145587 
>>145586
ну ты петух, написано же под названием раздела как.
https://2ch.hk/abu/res/35986.html
https://2ch.hk/abu/res/35567.html
Аноним 29/12/14 Пнд 20:45:34 #427 №145588 
>>145581
Тяжелых нет, легких сколько угодно. В основном из-за невесомости, особенно у первых космонавтов (Титов, Терешкова). У кого-то на Союзах случилось около-параноидальное состояние, в результате полет пришлось прервать. (не помню какой именно полет)
Аноним 29/12/14 Пнд 23:14:21 #428 №145623 
14198840613350.jpg
>>145585
Поляков. 437 суток на Мире.
Аноним 29/12/14 Пнд 23:38:10 #429 №145629 
>>140374
Значит ли это, что 5 миллиардов лет назад Проксима Центавра была на в разы меньшем расстоянии от нас, чем сейчас?
Аноним 30/12/14 Втр 00:16:20 #430 №145632 
>>145629
можно было пешком сходить
Аноним 30/12/14 Втр 00:17:49 #431 №145633 
Что будет, если на Землю налетит сверхмассивный пролапсар?
Аноним 30/12/14 Втр 00:26:47 #432 №145635 
>>145629
Нет.
>>145633
>пролапсар
Что-что налетит?
Аноним 30/12/14 Втр 00:29:33 #433 №145637 
>>145635
да, налетит
Аноним 30/12/14 Втр 11:08:23 #434 №145696 
>>145633
>сверхмассивный пролапсар
Может коллапсар? В любом случае, чтобы ты там не подразумевал, будет большой пиздец.
Аноним 30/12/14 Втр 11:12:34 #435 №145697 
14199271543200.jpg
Погуглил картинки пролапсара.
Аноним 30/12/14 Втр 13:00:33 #436 №145702 
какие могут быть проблемы при протекании беременности в невесомости?
Аноним 30/12/14 Втр 14:02:42 #437 №145707 
что будет с трехмерным обьектом (живым и неживым), попавшим в четерех, пяти - и более-мерное пространство??
Аноним 30/12/14 Втр 14:17:08 #438 №145708 
>>145702
У тебя беременность протекла.
Аноним 30/12/14 Втр 14:28:12 #439 №145710 
14199388929890.jpg
>>145708
Аноним 30/12/14 Втр 15:16:11 #440 №145724 
>>145702
Копротивление общественности
Аноним 30/12/14 Втр 15:37:45 #441 №145727 
>>145633
> пролапсар
Как же я проиграл.
Аноним 30/12/14 Втр 15:38:24 #442 №145728 
>>145707
Он спасёт человечество.
Аноним 30/12/14 Втр 15:58:59 #443 №145733 
>>145702
ХЗ, ничего. Только повышенный радиоционный фон может увеличить вероятность ненужных мутаций. Но дрозофилам норм, может и мышкам, не знаю.
>>145707
ничего. Мы все в 11 мерном пространстве. На самом деле не целиком На самом деле нет
Аноним 30/12/14 Втр 15:59:44 #444 №145734 
>>145733
>радиационный
быстрофикс
Аноним 30/12/14 Втр 16:16:07 #445 №145735 
>>145733
> ХЗ, ничего.
ХЗ, иди нахуй. Механизм переворачивания плода - хз. Роды тоже на орбите принимать собрался, хз? Или может во время спуска с орбиты, на седьмом месяце воды отойдут - а мы смеемся, хз?
Да даже радиационного фонда достаточно. Вот ты отправишь свою тяночку на орбиту, чтобы она родила тебе в ночь не то сына не то дочь?
Не мышонка, не лягушку, а стивена хокинга? Вот и она не захочет.
Аноним 30/12/14 Втр 16:23:59 #446 №145736 
>>145735
допустим сделали совершенную радиационную защиту
Аноним 30/12/14 Втр 16:51:44 #447 №145737 
>>137537
Чем отличается квантовая физика от физики элементарных частиц?
Аноним 30/12/14 Втр 17:15:10 #448 №145740 
>>145736
Допустим, что плод не будет нормально развиваться в условиях невесомости, здоровых мужиков через полгода на руках вытаскивают из капсулы.
Аноним 30/12/14 Втр 17:17:28 #449 №145741 
>>145740
можно оставить его жить на орбите
Аноним 30/12/14 Втр 17:19:28 #450 №145743 
>>145737
Два подвида квантовой физики
Один описывает описывает явления, которые происходят на квантовом уровне, второй изучает структуру элементарных частиц и их взаимодействия.
Аноним 30/12/14 Втр 17:20:19 #451 №145744 
>>145741
Лучше сразу в шлюз выбросить, родится кусок кровавого творога.
Аноним 30/12/14 Втр 17:25:09 #452 №145746 
>>145743
Вот это мне как раз не очень понятно. Разве элементарные частицы сейчас описываются не квантовой физикой?
Что из этих двух ветвей физики более общее, а что частное?
Аноним 30/12/14 Втр 17:35:04 #453 №145748 
>>145746
Квантовая механика более общая часть. Она описывает явления микромира. ФВЭ занимается исключительно частицами, например уебыванием их в ускорителе и наблюдений что из этого выйдет и насколько то, что получилось совпадает с теорией, например.
Аноним 31/12/14 Срд 00:13:45 #454 №145798 
14199740259820.jpg
Анон, помоги разобраться. Все мы знаем, что первыми в космос смогли совки, запустив Спутник-1 в 1957м году, но только что пролистывав свою старую энциклопедию, я наткнулся на след. строки:«Важнейший шаг в развитии ракетной техники был сделан еще в 1949г., когда на испытательном полигоне Уайтсендз в штате Нью-Мексико с носа ракеты Фау2 была запущена маленькая ракета «WAC-Корпорел». Она достигла рекордной для того времени высоты 393км и скорости 8286км/ч.»
То есть выходит, что эта ракета была за условной границей космоса (100км). И соответственно должна считаться первонахом-железякой, или я в чем-то решительно туплю?
Аноним 31/12/14 Срд 00:17:54 #455 №145799 
>>145798
100 км только у сасш граница.
Аноним 31/12/14 Срд 00:28:38 #456 №145800 
>>145799
У всех белых людей
Пруф - международное право
Аноним 31/12/14 Срд 00:30:45 #457 №145802 
>>145798
https://ru.wikipedia.org/wiki/Искусственный_спутник_Земли
Аноним 31/12/14 Срд 00:36:15 #458 №145807 
>>145798
Считается орбитальный полет, а не суборбитальный.
Аноним 31/12/14 Срд 00:37:26 #459 №145808 
>>145800
В любом случае это не спутник. Спутник летает по орбите.
И кстати первой по суборбите прошла Фау-2 еще в 1944. Высота 188 км.
Аноним 31/12/14 Срд 00:39:57 #460 №145811 
>>145807
Довольно странно, все-таки полет в космос как никак. А можешь накидать чтива про другие суборбиталки старше 57го?
Аноним 31/12/14 Срд 01:21:29 #461 №145813 
>>144155
>однажды приложенная сила продолжает действовать
т.е. если я приложу некую силу, которая придаст телу ускорение 1 м/c^2, то тело будет бесконечно ускорятся?
Аноним 31/12/14 Срд 01:32:50 #462 №145815 
>>144516
>можно покинуть, двигаясь на двигателях с постоянной скоростью, например 100км в час.
Нельзя. Во-первых что значит двигаясь на двигателях? Если ты имеешь ввиду, что двигатели всегда будут включены, то как может идти речь о постоянной скорости? Если ты хочешь лететь все время по направлению от земли "вверх" с постоянной скоростью на включенных движках, то в момент когда ты будешь покидать орбиту земли твоя скорость в апогее будет условные 100км/ч, но в перигее ты превысишь вторую космическую.
Аноним 31/12/14 Срд 01:56:52 #463 №145818 
>>145815
Неважно. Смысл его поста в том, что чисто теоретически можно выехать из сферы Хилла планеты, не набирая первую космическую, пусть это и будет пиздецки затратно.
Аноним 31/12/14 Срд 02:00:53 #464 №145819 
>>145813
нет, оно будет бесконечно лететь со скоростью 1 м/c^2 пока на него не действуют другие силы
Аноним 31/12/14 Срд 02:08:58 #465 №145820 
>>145819
Ясно. Я просто решил доебаться к словам. Правильная формулировка всегда важна.
Аноним 31/12/14 Срд 02:13:42 #466 №145821 
14199812225830.jpg
>>145820
малайца
Аноним 31/12/14 Срд 04:16:29 #467 №145831 
14199885897190.jpg
>>145819
> со скоростью 1 м/c^2
Аноним 31/12/14 Срд 05:01:09 #468 №145834 DELETED
>>145798

Американы не смогли сделать "пи-пи-пи". Соснули у СССР в микроэлектронике.
Аноним 31/12/14 Срд 07:56:55 #469 №145840 
>>145819
Ловите наркомана.
>>145815
Какая еще орбита? Единственная орбита на которую он таким образом выйдет - геостационарная, все, что ниже будет считаться суборбитальным полетом.
два вопроса Аноним 31/12/14 Срд 11:51:04 #470 №145860 
14200158643980.jpg
I. Што будет с Дейвом, когда на космической станции запустят вращение для создания искусственной силы тяжести, а он в это время не будет ничего касаться? Станция раскрутится, а он будет с ужасом наблюдать проносящиеся мимо него переборки, пока одна из них не уебёт его по башке?

II. Насколько вероятно, что с той стороны Солнца есть ещё одна "Земля" или другое небесное тело, которое вращается вокруг Солнца с той же скоростью, что и наша Земля, и именно поэтому мы не можем увидеть это небесное тело?
Аноним 31/12/14 Срд 12:25:08 #471 №145863 
>>145860
1. Грубо говоря да. Но станция не за секунду до 7200об/мин будет разгоняться, и воздух внутри есть, так что эффект малозаметен будет, его скорее к полу прибьет.

2. Невероятно, т.к. мало того что это противоречит всем теориям планетообразования, так гравитационное влияние такого тела было бы обнаружено задолго до прямых наблюдений. Осенью была работа в Nature, намекавшая на потенциальное существование "суперземли" где-то в облаке Оорта, косвенно оказывающей влияние на объекты типа Седны через эффект Козаи. А ты про землю на каких-то 1 AU от Солнца.
Аноним 31/12/14 Срд 12:31:08 #472 №145865 
>>145863
Пока что это построения уровня Вулкана.
Аноним 31/12/14 Срд 14:24:26 #473 №145872 
>>145860
ДАЭТОЖ НИБИРУ
Аноним 31/12/14 Срд 15:20:04 #474 №145879 
Анон, вопрос по теме игор. Есь ли какие то игори о жизнь/выживании на косм станции? Вроде Space Station 13, но синглплеер. Чтобы вокруг космос, выращивать еду, копать астероиды и заряжать изотопами реактор, ремонтировать солнечные панели и т.д
Аноним 31/12/14 Срд 15:40:17 #475 №145882 
>>145879
чет про марс есть, ща погоди
вот Mars Colony Challenger
http://rutracker.org/forum/viewtopic.php?t=4556865
Mars Colony: Challenger - комплексный симулятор по строительству и обслуживанию базы на поверхности Марса!
Вам придется управлять и поддерживать полную работоспособность всех агрегатов и устройств жизнедеятельности. Следить за давлением воздуха, выращивать растения, протягивать коммуникации, и добывать ресурсы, без которых база не сможет существовать самостоятельно.

Игра предлагает три зоны, каждая из которых имеет различные задания. Каждая миссия состоит из 7 фаз. На каждой фазе вам будут даваться новые задания по расширению базы.

Можно играть с друзьями онлайн.
Аноним 31/12/14 Срд 16:32:48 #476 №145885 
>>145879
В игори не играю, но "Марсианина" Вейра читни. Не игорь, но ты прям такой реквест состряпал, что прям мимо не пройти.
Аноним 31/12/14 Срд 17:48:09 #477 №145890 
>>145879

Там ниже по нулевой висит космических инженегров-тред, посмотри, может - то, что тебе нужно.
Аноним 31/12/14 Срд 18:26:05 #478 №145893 
14200395654890.png
14200395655091.png
14200395655272.png
14200395655633.png
>>145879
http://mars-sim.sourceforge.net/
100% реали аутизм, вроде Авроры, только про колонию.
Аноним 31/12/14 Срд 18:29:48 #479 №145894 
14200397884090.gif
14200397884111.gif
14200397884132.gif
>>145879
Ещё Outpost. 100% твердота, уникальная игра в своем духе. У него есть сиквел Outpost 2, но тот более фантастичный и обыкновенный.
Аноним 31/12/14 Срд 21:58:13 #480 №145898 
>>145840
Насчет суборбитального полета согласен, орбиты вокруг Земли не получится (хотя если теоретизировать и рассматривать Землю как материальную точку...). А вот насчет геостационарной орбиты не понял, если достаточно отлететь от Земли, то в итоге все равно перейдешь на орбиту вокруг Солнца. Да и почему полет ниже ГСО считается суборбитальным?
Аноним 31/12/14 Срд 21:58:33 #481 №145899 
>>145894
Огоблин) Я играл в это когда-то. Пытался точнее. Нихкяшеньки не поня. Там манулы длиннее кода должны были бы быть.
С НОВЫМ ГОДОМ! Аноним 01/01/15 Чтв 12:49:10 #482 №145928 
14201057509660.jpg
Поздравляю с Новым годом, спейсач! Завсегдатаи вопросотреда, телескопщики, работники около- и космических отраслей, порашники, заговорщики, кефирщики, а также огурцы, орбитерщики, инженегры и прочие уважаемые анонимные любители космических полётов, а также мод-кун и тех спейсачеры, которых забыл упомянуть - поздравляю всех вас!
Аноним 01/01/15 Чтв 13:45:55 #483 №145929 
Доброго времени суток, уважаемые Фон Брауны.
Я учусь в одной из топовых ракетостроительных шараг в стране, на кафедре, одно из направлений которой, как раз тесно связано с космическим машиностроением. Однако, на данный момент, все мои познания о космосе ограничиваются десятком часов ксп и кинчиком с МакКонахи. У меня есть безумная идея полететь на мкс пойти работать на какую-нибудь Энергию или что-то вроде того, но не хотелось бы, чтобы единственной моей мотивацией было космас - это прикольно, поэтому такой реквест: посоветуйте годной литературы обо всем сразу, так сказать обзор всего, что связано с космосом в целом и космическим машиностроение в целом. Хотелось бы иметь хоть сколько-нибудь обширное адекватное реальности представление картины в целом, чтобы ясно было какая из областей мне интересна более остального, куда развиваться и идти работать. Не то, что бы я боюсь читать, но было бы здорово, если пришлось бы прочесть всего-лишь несколько книг (обо всем в общем и ни о чем конкретно), нежели целую библиотеку.
И еще конкретный вопрос: как широко используются в космосе турбодетандеры?
Аноним 01/01/15 Чтв 13:48:32 #484 №145930 
Забавно пикрелейтед >>137590 - выпускник моей кафедры. :3
>>145929 - кун
Аноним 01/01/15 Чтв 14:55:39 #485 №145936 
>>145929
>Я учусь в одной из топовых ракетостроительных шараг в стране
Ананасик, ответь на пару вопросов профану
Сколько лет учиться? Реально ли потом работать по профессии? Ну и достойная ли будет зп?
Аноним 01/01/15 Чтв 15:55:38 #486 №145939 
>>145894
>>145893
А именно косм. станцию глубоко в космосе, далеко от людей, а не анальные колонии?
>>145890
Не то. Там нет выживача
Аноним 01/01/15 Чтв 16:10:16 #487 №145940 
>>145929
Черток "Ракеты и люди".
https://captcha.yandex.net/image?key=20g6xuE2GR67JgdwTMfty4qlxF3zXnsB
Аноним 01/01/15 Чтв 16:12:11 #488 №145941 
>>145940
404 Not found
requested captcha image has expired
Аноним 01/01/15 Чтв 16:54:04 #489 №145943 
>>145941
http://militera.lib.ru/explo/chertok_be/index.html
Аноним 01/01/15 Чтв 17:06:46 #490 №145945 
слышал, что скорость света постоянно немного меняется? как такое возможно? меня учили что это константа
Аноним 01/01/15 Чтв 17:10:46 #491 №145946 
>>145945
В военное время может достигать 900 тыс. км/сек.
в среде и в вакууме разная
Аноним 01/01/15 Чтв 17:14:23 #492 №145947 
>>145946
https://ru.wikipedia.org/wiki/Переменная_скорость_света
поясните аутисту
"Идея Моффата и команды Альбрехта — Магейжу состоит в том, что свет распространялся на целых 60 порядков быстрее в ранней Вселенной, таким образом, отдалённые области расширяющейся Вселенной успели повзаимодействовать на начальном этапе Вселенной."
Аноним 01/01/15 Чтв 17:31:53 #493 №145952 
Из-за вас, мне сегодня приснилось, что у меня друг, которого я давно не видел упоролся 15-мерными гиперкубами, построил несколько 15-мерных комнат и повесился в одной из них.
С Новым Годом.
Аноним 01/01/15 Чтв 17:32:52 #494 №145953 
>>145952
охуеть, пили кулстори
Аноним 01/01/15 Чтв 17:39:11 #495 №145955 
>>145953
Что пилить-то? Сон же. Да и то мне в нем об этом рассказали с минимумом иллюстраций.
На самом деле к этому треду это имеет косвенное отношение, но в одном из других вкидывали нерешенные математические задачи среди которых была задача про 7-мерный гиперкуб.
Аноним 01/01/15 Чтв 18:43:01 #496 №145967 
>>145952
>упоролся 15-мерными гиперкубами, построил несколько 15-мерных комнат
Недавно про такого кино сняли. Джозефом Купером звали.
Аноним 01/01/15 Чтв 22:14:48 #497 №146007 
>>145936
Ну смотря где, на спеца ли, на бака\магистра. Работать по спецухе чаще всего вполне реально, но либо все в говне и проебано, либо за 15к. Тебя конкретно что интересует-то?
Аноним 01/01/15 Чтв 23:53:42 #498 №146035 
>>146007
Просто хочу понять, к примеру отдавать ли ребёнка в ракетостроение или лучше на баналього юриста
Аноним 02/01/15 Птн 00:17:17 #499 №146039 
>>146035
Смотря, опять же, от поставленных целей.
Лучше, очевидно, туда, куда ребенок сам захочет. Учиться на юриста, если душа к оному хоть сколько-нибудь не лежит, ровно в той же степени тяжко и ненавистно, что и на инженегра. С другой стороны, не знаю как там у юристов всяких, но наши выпускники в подавляющем большинстве работают не в нии да кб, а вполне достойно зарабатывают на смежных и вовсе не смежных отрослях, да хоть на продажах своей продукции, если это, конечно, не какие-нибудь "ракетные двигатели". Впрочем, это все общие слова и философия, не совсем к теме доски относящаяся.
Аноним 02/01/15 Птн 00:31:59 #500 №146040 
>>146035
Лучше на программиста, в какую-нибудь хорошую альма матер. Тогда он сам выберет себе будущее.
Аноним 02/01/15 Птн 00:46:08 #501 №146041 
>>146039
Анон, не философствую молю. Я сам прекрасно понимаю что прежде чем говорить малаца спайцану нужно заинтересовать его. Я просто хочу понять, оплачиваемая ли это профессия
Аноним 02/01/15 Птн 10:45:13 #502 №146056 
>>146041
Скорее нет, чем да.
Если закончить что-то менее узкое, чем "ракетные двигатели", то всегда будет возможность заработать на второй работе\каких-нибудь леваках. Ну а так, в плане заработка, как мне кажется, эта отрасль может предложить что-то сладкое только в больших перспективах.
Я тебе так скажу, с группы дипломников кафедры защиты РКА и СЖО только один студент пошел на Энергию да и тот, потому что целевик.
Аноним 02/01/15 Птн 13:47:31 #503 №146062 
>>137537
Анон, а если взять коробку, всунуть туда фонарик на радиоуправлении и запаять это. Если мы включим дальше фонарик и отключим его то получим ли мы коробку в которой будут хранится фотоны?
Аноним 02/01/15 Птн 13:54:21 #504 №146063 
>>146062
Получим. Только на очень короткое время.
Аноним 02/01/15 Птн 14:45:36 #505 №146066 
>>146062
конечно получим. чем дольше прогорит фонарик - тем больше фотонов. только когда будешь коробку открывать, не смотри туда или используй сварочные очки - иначе рискуешь ослепнуть когда фотоны вырвутся на свободу
Аноним 02/01/15 Птн 14:54:55 #506 №146068 
>>146066
тут главное не перестараться, ибо при превышении критического давления фотонов на м2 стенок ящика, получим вполне себе ядерную бомбу на 1-2 килотонны
Аноним 02/01/15 Птн 15:50:23 #507 №146071 
>>146063
>Только на очень короткое время
А если стенки коробки изнутри обложить зеркалами?
Аноним 02/01/15 Птн 16:04:11 #508 №146072 
>>146062
>всунуть туда фонарик на радиоуправлении
Радиоволны и есть фотоны. Фотоны - кванты электромагнитного излучения. Если фотоны могут попасть в коробочку снаружи, то что помешает им ее покинуть?
Аноним 02/01/15 Птн 16:25:25 #509 №146074 
>>146072
Хорошо, тогда фонарик с таймером в радионепрозрачной коробке с зеркалами на стенках.
Аноним 02/01/15 Птн 16:42:05 #510 №146076 
>>146074
зеркал со 100% отражением ИРЛ не существует
Аноним 02/01/15 Птн 16:50:04 #511 №146077 
>>146076
А у нас будет теоретический случай.
Аноним 02/01/15 Птн 19:15:55 #512 №146085 
>>146077
Теория этого тоже не позволяет.
По разным причинам, например существование давления света накладывает запрет на 100% отражение. Закон сохранения массы и энергии, дорогой.
Аноним 02/01/15 Птн 20:54:22 #513 №146095 
>>146085
ОК, как долго свет будет затухать в таком устройстве?
Аноним 02/01/15 Птн 23:09:15 #514 №146109 
>>146095
Намного быстрее чем ты успеешь подумать об этом.
Аноним 07/01/15 Срд 23:37:48 #515 №147211 
Прочитал в унылейшем "марсианине" Уира о том, что Марсианская долина образовалась за один день. Гугл молчит. Это правда? Если да, то дайте ссылку.
Аноним 23/03/15 Пнд 22:55:20 #516 №162625 
>>146088
Перекат.
comments powered by Disqus

Отзывы и предложения